Example Questions from students

Lakukan tugas rumah & ujian kamu dengan baik sekarang menggunakan Quizwiz!

AB was diagnosed with OCD 2 weeks ago because before he leaves his house he has to check multiple times that all the doors and windows are locked and that all the appliances are turned off in his home. He even has to drive by his house multiple times before he leaves to go to his destination and has to periodically check up on his home while he is gone to make sure that no one is trying to break in. AB is going to his first treatment appointment for his OCD, what would be the most appropriate and effective first line treatment that AB would get? Select all that apply. A. A type of CBT called Exposure and Response Prevention B. Clinical Trial C. Medication D. Joining a support group

A &C, International OCD foundation states that the most effect treatment for OCD is CBT and/or medication. The type of CBT that is most effective for OCD is exposure and response prevention. While joining a support group may be beneficial for a person, it would not necessarily treat their OCD diagnosis. Also, joining a clinical trial would not be the first way a physician would try to treat OCD since the strongest evidence suggests that CBT is the best treatment.

Long term use of antipsychotic medications is associated with which of the following, choose all that apply: A. Metabolic syndrome B. Insomnia C. Increased risk of drug-induced movement disorders D. Cardiovascular, hepatic, and immune system problems E. Impaired skin integrety

A, C, and D.

June is in the ED while experiencing a psychotic episode. She has a plan to kill herself. Which method of suicide puts June at the highest risk for following through with her plans? A) She plans to ingest a full bottle of medication. B) She plans to shoot herself with her husband's hand gun. C) She plans to cut her wrist. D) She plans to walk out in freezing weather and wait to die.

B Rational: According to Halter, Varcarolis' Foundations of Psychiatric Mental Health Nursing - 8th edition, high risk methods are called "hard methods". These methods tend to be far more effective and ending a person's life. These include: using a gun, jumping off high locations, hanging, and other methods. Soft methods (lower risk) include: cutting, inhaling natural gas, and ingesting pills. The best answer is B- because hard method plans are much more likely to effectively end a person's life. They are all high risk but B is highest risk plan. A C and D are soft methods, B is a hard method

Which example is the BEST way a nurse could approach a physician or other healthcare professional when addressing a mental health patient? A. "The patient in room 454 is crazy. She will not answer my questions! She just sits there with a blank look on her face." B. "There is an extremely depressed female in room 454. She does not wish to speak to me and will not cooperate." C. "The patient in room 454 is either sad or depressed but will not tell me why and asked me to leave. I refuse to go back in and help her." D. "The female patient in room 454 is showing signs of depression. This morning when I went in for her daily assessment she refused to answer questions or speak with me, this is the 3rd day this has happened. It also looks like she has been crying." E. "The depressed patient in room 454 refused to answer my questions during this mornings assessment. I don't think we can help her anymore."

D Rationale: Patients even when they are unwilling to cooperate should be addressed respectfully. D is the best answer because the patient is being identified as a human being rather than her illness. The nurse in answer D also went on to explain as to why she thinks the patient is depressed rather than assuming based on their interactions. If the patient was just having a bad morning because she did not sleep well, that does not make her depressed. All the other answers contain something that is disrespectful. You should never call a patient crazy, or assume she has a clinical diagnosis of depression without a physician. You should also not give up on a patient, if you wish to have another nurse attempt to talk to the patient there are better ways to do this.

You have a patient that is on Topiramate and you recently got back their latest lab information. The CCr <70 ml/min/1.73m2, and the physician has order a does reduction knowing that the medication is affect which organs in the body? A. Hepatic organs B. Repiratory organs C. Reproductive Organs D. Intestinal organs E. None of the above.

E, none of the above. CCr is Creatine Clearance rate, and this drug is affect the kidneys and their ability to filter the medication out of the body.

What populations are at the highest risk for delirium? (choose all that apply) A. ICU patients B. Teenagers C. elderly D. 20 year old with HTN

The correct answers are A. and C. Most patients who suffer from delirium are patients who are terminally ill (80-90% of delirium cases) and the elderly (about 20% of pts 65 and over) are affected with delirium. (Beware of Delirium, Tucker and Ferry).

Tommy is a 22 year old male who you are assessing during a yearly physical. During your interview, Tommy is constantly sighing and talking about how stressful his college classes are, often saying things like "I just can't anymore". A priority nursing diagnosis for Tommy would NOT be (select all that apply): A. Severe Anxiety related to unmet needs B. Acute Pain related to emotional injury C. Hopelessness related to long term stress D. Social Isolation related to inability to engage in satisfying personal relationships

A, B, and D are the correct answers due to the presentation of signs and symptoms and related factors outlined in the Pearson Nursing Diagnosis Handbook with NIC Interventions & NOC Outcomes (Wilkinson, 2014, pg. 367). C is incorrect because that is the most appropriate NANDA for this situation.

XX is a 41-year-old female patient who was diagnosed with depression 8 months ago. After this, her psychiatrist started her on fluoxetine (Prozac). After a few months being at home with her husband, keeping to a strict medication schedule, and spending time with her grandchildren, XX told her psychitrist that she felt back to her old self. After this revelation, she abruptly stopped taking her Prozac. Which of the following symptoms would you expect XX to experience after this discontinuation of medication? (Select all that apply): A. insomnia B.vivid dreams C. angioedema D. vomitting

A, B, and D. When prescribed an SSRI, such as Prozac, patients are instructed to not abruptly discontinue their antidepressant medication. Stopped usage can cause anticholinergic rebound symptoms known as SSRI withdrawal syndrome, which can cause flulike symptoms. These symptoms can last from one to seven weeks, and can be dangerous if suicidal idealation is present. It is important for nurses to recognize this so they can be cared for appropriatly, and watched closly for suicidal tendencies that can be fatal. (pg. 323, Giddens, Concepts for Nursing Practice, Second Edition 2017)

A patient is admitted to the mental health unit at the hospital with a diagnosis of major depressive disorder. Which of the following clinical manifestations would the nurse expect with this diagnosis? (Select all that apply) a. Feelings of worthlessness b. Inability to concentrate c. Hyperactivity d. Persistent thoughts of death or suicide e. Rapid speech

A, B, and D. According to Halter, common clinical manifestations of major depressive disorder are feelings of worthlessness or hopelessness, persistent thoughts of death or suicide, and an inability to concentrate or make decisions. There are also others such as anhedonia, fatigue, sleep disturbances, change in eating habits, and change in physical activity. The other manifestations listed (C and E) commonly occur in individuals who are manic (Halter, 2014, pg. 233, 250).

John, a 35-year-old male, was just diagnosed with schizoaffective behavior and is currently in the manic phase and admitted to your unit. He is unsure how the doctors made his diagnosis. What are some positive symptoms of acute manic phase that the doctor would've been looking for? (select all that apply) A. A decreased need for sleep B. Experiencing anergia (lack of energy) and avolition (lack of motivation) C. Flight of ideas/rapid thinking D. Pretentiousness (modest) behavior

A and C. Answer A is correct because a person in acute manic phase is so stimulated and aroused that they can go for days without sleep. Answer C is correct because they are again so stimulated that their thoughts keep racing and are unable to stay on one topic for long periods of time. They are easily distracted, often by making "clang associations" - the rhyming of words. Answer B is not correct because lack of energy and motivation occur during the depressive psychosis rather than the manic psychosis. Answer D is incorrect because a person experiencing manic psychosis will have a grandiose behavior instead of a humbled behavior. (Wilkinson, 2016) pg. 1028-1029. - NANDA BOOK

If your patient is experiencing symptoms of severe/panic levels of anxiety, what are appropriate interventions that should be carried out by the nurse? Select all that apply. a. Reduce the amount of environmental stimuli b. Continue to ask the patient why they are experiencing anxiety c. Maintain a calm manner d. Leave the patient to go find help

A and C. By reducing the amount of environmental stimuli, the nurse is able to minimize further escalation of the patient's anxiety. The nurse must maintain a calm manner because anxiety is communicated interpersonally. If the nurse is not calm, it will be difficult for the patient to remain calm. B is not the correct answer because a person who is experiencing anxiety has difficulty concentrating and processing information. D is not the correct answer because leaving them alone will make the patient feel abandoned (Halter, pg. 296).

Mrs. Peters experiences feelings of reduced need for sleep, and racing thoughts. A student nurse comes in to assess Mrs. Peters and finds that she can not get a word in to ask questions, Mrs. Peters is loud and talks all about how she is going to do so many things when she gets out of the hospital. What can be expected with Mrs. Peters? choose all that apply. A is experiencing high levels of serotonin, norepinephrine, and dopamine. B is a classic example of bipolar 2 c would be expected to take lithium. D, she experiencing high levels of serotonin, Gaba, and dopamine. E all of the above.

A and C. serotonin, norepinephrine, and dopamine are all in high levels in bipolar disorder during mania, Gaba slows the processes of the brain and would not have this impact on bipolar 1 disorder. bipolar 2 only ranges from depression to hypomania and this is more of a manic state.

The nurse starts her shift and is about to begin her usual head to toe assessment on a 14 yr. old male that just came into the unit. The nurse understands that adolescents are fearful of whatever the nurse finds is reported back to their parents. While assessing the patient the nurse remembers to conduct a HEADSSS Psychosocial Interview. The nurse discovers the boy is restless, is rambling on about seeing a man shoot another man (but the nurse sees nothing) and looks like he has baggy eyes. What does the nurse suspect might have caused this acute confusion according to what she knows about the HEADSSS Psychosocial Interview risk factors: (Select all that applly) The Home environment (e.g., relations with family members) Education (e.g., school performance) Activities (e.g., sports participation, after-school activities, peer relations) Drug, alcohol or tobacco use Sexuality or Suicide risk or symptoms of depression or other mental disorder or Safety of the patient (e.g., how safe does the patient feel at home and school)

A, B, C, D. The answer is A because tensions in the family could have caused this patient to make certain decisions to could have caused this presentation to unfold. The answer B because we don't know to any extent if the patient's school performance has caused this behavior or current condition. We don't have enough information about the patient's history to know what might have caused this presentation of signs and symptoms. The answer D because drugs, alcohol or tobacco use could have made the patient restless and even see and hear one man shoot another man, that no one else can see and even, caused the patient some sleep deprivation. The answer E because until we get more patient history, we can't rule out sexual activity playing a role, or whether or not this was a suicide attempt or S/S of a mental disorder. We definitely, don't know how safe this patient is with or without other people around. According to (Varcarolis & Halter, 2018) and Acute Confusion NANDA section in (Wilkinson, Barcus, & Wilkinson, 2017).

B.J. is a 78-year-old female that is in the hospital due to a hip replacement. She has a history of Dementia, Hypertension and Diabetes. She wears hearing aids and glasses daily. She has been on the Ortho-surgical unit for the past 5 days and has increasingly gotten more confused. The family is concerned and wants to know what is going on. The nurse knows that delirium can be caused by several different things. Which of the following should the nurse pay attention to as probable causes? Select all that apply: A.Infection B. Post-Surgery Medication C. Hypoxia D. Lack of Family Involvement in Care E. Pre-Existing Dementia

A, B, C, E. As we talked in class having a pre-existing mental impairment puts you at more of a risk, as well as opioids are generally given after a surgery such as the hip replacement. In the book it states that the causes of delirium can be many things (IWATCHDEATH).

You are the primary nurse in a mental health unit of the hospital. A new patient has just arrived to be admitted voluntarily. The patient is being admitted with a diagnosis of major depressive disorder and is currently experiencing strong suicidal ideation. On your initial assessment, which of the following are KEY assessment findings related to the major depression diagnosis? (Select all that apply): a. Chronic pain b. Anhedonia C. Anxiety D. Hypomanic State

A, B, and C. According to Halter in Varcarolis' Foundations of Psychiatric Mental Health Nursing, several key symptoms to assess with major depressive disorder include: depressed mood, anhedonia, anxiety, and chronic pain among many others (Halter, 2018, p. 251). When a patient has had at least one major depressive episode and at least one hypomanic episode together, the likely diagnosis is bipolar II disorder, NOT major depressive disorder (Halter, 2018, p. 223).

Mrs, Peters comes in for a checkup visit 10 days later to get labs and blood work done. she is complaining of feeling like she needs to throw up, she has been taking herb to try and calm her stomach called st johns wart. and she is laying down with her hands on her head saying "the world is spinning" when you walk in. is carrying a gallon of water with and has a shakiness to her hands when you take her pulse. this is due to what level of drug in her system. A. Lithium: 0.1 - 1.0 mcg/ml B. lamotrigine: 4.0 - 6.0 mcg/ml C. Lithium: 1.5 - 2.0 Meq/ml D. Lamotrigine: 1.2 - 3.5 Meq/ml

A. that is the toxicity range, she is explaining feelings of nausea vomiting, Dizziness lethargy, and fine tremors. it takes 7-14 days for lithium to build up in the system

Tyler is an 6 year old child with cynophobia (fear of dogs), trypanophobia (fear of shots), and ophidophobia (fear of snakes). While many children grow out of these types of phobias, Tyler is scheduled to have his booster shots in your clinic this week. His mother is concerned that his fear of shots will negatively impact the care he gets from your facility as he has been a difficult patient in the past because of this fear. She wants to schedule additional time with Tyler to speak about his fears, especially of shots, prior to receiving any. Which of the following are the best interventions for Tyler's fear(s)? (Select All That Apply). A) Assess why Tyler has the fears he has, especially those that are experience based. Correct any misconceptions he may have about items he fears B) Help Tyler confront his fears in the controlled environment of the office C) Have Tyler's mother step in for Tyler every time he gets fearful to manage his anxiety D) Provide Tyler with the tools to enhance his coping mechanisms - such as meditation, physical activity, relaxing hobbies (coloring, reading, etc.)

A, B, and D Rationale: A is the easiest, short term, intervention for Tyler. Perhaps he believes that getting shots will hurt him or otherwise make him sick/unable to do something he wishes to do. If the nurse can listen to Tyler's perception of the fear, (s)he can clear up any misconceptions he may have. He may have seen a needle that would be used to inject into bone and became afraid that all needles were that size. B is also a quick intervention. If the nurse can show Tyler that he will be safe in the office to face his fears, and he can see others receiving shots he may feel more confident to face his fear of them. One way to do this would be to have the nurse who would be doing his injections give one to his mother (booster, flu shot, normal saline, etc.) depending on what she needed/preferred. This would show him that someone he trusts (his mother) has just had a shot and didn't have any of the problems he may associate with them. This will build on changing his perceptions of the trypanophobia. D is a longer intervention that will be built over a period of time. Coping strategies are always helpful for individuals of any age, and teaching Tyler now that he can control how he feels about his fears and calm himself down will be helpful for him in the future. For example, if he knows that changing his breathing pattern before/during a shot will decrease his anxiety and fear, he can transfer that knowledge to his other fears. C is not a correct answer. Even though many children grow out of these fears, it is important that Tyler learn how to cope and handle his fears for himself. If his mother stepped in every time he got anxious or fearful and handled it for him, much like how D affected the future, so too would this. Tyler would learn that if he said he was afraid, his mother would take care of it for him. This would decrease his ability to cope and overcome.

A patient diagnosed with obsessive-compulsive disorder (OCD) carries hand soap everywhere she goes. She excessively scrubs her hands every time she touches surfaces, objects, and people. The nurse understands that the patient's behavior is an attempt to accomplish which of the following: a. promoting good hand hygiene b. relieve anxiety c. avoid interacting with others d. All of the above

B. - CORRECT. OCD is an anxiety disorder in which people with this condition act on their thoughts in attempt to relieve anxiety. In this case, the pt is obsessed with keeping free of bacteria that she walks around with soap and hand washes excessively to have a piece of mind that she is germ-free.

Which statement made by the patient would indicate to the working nurse that they are experiencing a delusion? A. "I am going to cover my walls with aluminum foil to block the aliens from getting in and controlling my thoughts." B. "I see a dark man in the corner of my room almost every night." C. "I hear voices that tell me to punch my fist through the wall." D. "I constantly feel spiders crawling up and down my arms and I can't take it."

A Rationale: (A) is the correct answer. Delusions are a false belief held regardless of evidence to the contrary. Answer (A) represents a control delusion. This is defined as believing that another person, group of individuals, or external force controls thoughts, feelings, impulses, or behavior. Answer (B) represents a visual hallucination, seeing people or things. Answer (C) represents an auditory hallucination, or hearing voices or sounds. Answer (D) represents a tactile hallucination, or feeling bodily sensations.

Lydia was recently diagnosed with type II bipolar disorder and prescribed venlafaxine (Effexor) and olanzapine (Zyprexa) and is typically hypomanic. After starting the medications, Lydia has never felt so energized, she is extremely elated and feels on top of the world. She decided life is too short to sleep and stays awake as much as possible. She picked up new hobbies such as going on shopping sprees every weekend and vacationing often. Ever since she started her medications, she became hyper-sexual and is rarely tired. Lydia's new motto in life is to "say yes to everything, no matter how crazy it is". She does not feel the need to think things through. The most appropriate nursing diagnosis for her is: A. Poor impulse control r/t sleep deprivation B. Ineffective coping r/t alteration in cognitive function C. Disturbed personal identity r/t psychiatric disorder D. Self care deficit r/t bipolar disorder

A SNRIs increase the energy of a patient. In addition, when a patient in a hypomanic state does not sleep they can become manic and impulsive. This includes excessive spending, becoming hyper-sexual, and out of touch with reality. "Being manic means being busy during all hours of the day and night...excessive spending, use of credit cards, and high living continue even in the face of seriously depleted resources" (Halter 228). Due to this, impulse control is the priority diagnosis. Answer B is incorrect because she is not behaving this way as a means of coping, it is a side effect of her medication. Answer C is incorrect because she still maintains a self perception, she is just in a hyperactive state. Answer D is incorrect because even though she may be in a self care deficit, it is not the most appropriate answer. Also, the 'r/t bipolar disorder' is a medical diagnosis and cannot be used.

A nurse believes that one of her long-term care patients is at risk for social isolation. She has been making almost no eye contact, has a flat look on her face, and doesn't come to group activities anymore. Which of the following Interventions would be appropriate to apply? (Select all you can apply) A.Promotion of family cohesion and unity B.Encourage therapeutic play/group activities C. Only engage in conversation if the patient wants to D.Encourage an alcoholic drink because "it'll make her more outgoing"

A and B are correct. They are listed in the Nanda book under interventions for social isolation and will encourage a more outgoing and social aspect of their life. C and D are obviously not therapeutic and you should never encourage an alcoholic drink. (Wilkinson, 2014, pg. 716).

John has been waking up in the middle of the night from recurring nightmares. He has had these nightmares ever since he returned from the Vietnam War. He startles easily at noises and occasionally has flashbacks to his combat experiences. Which of these may be contributing to John's PTSD? A. Hyperactive amygdala B. Hyperactive frontal lobe C. Decreased GABA D. Decreased acetylcholine

A and C because the amygdala has become hypersensitive and over reacts to stimuli. The amygdala alerts the brain to danger and stimulates the sympathetic nervous system, but causes an exaggerated response in individuals with anxiety or PTSD. The frontal lobe facilitates judgement and impulse control. GABA is a relaxing neurotransmitter so if there is a decrease in it, his anxiety and PTSD symptoms will be heightened. Acetylcholine functions through muscle movement and plays a role in dementia, but not PTSD. References from Halter's Foundations of Psychiatric Mental Health Nursing.

Mikey is a 13-year-old male that has been diagnosed with obsessive-compulsive disorder. This started two years ago when he was bullied by his peers in school and stuffed into a garbage can in the lunch room. After this incident, Mikey coincidently developed a terrible cold which he blamed on being put in a garbage can. Since then, Mikey continuously thinks about all of the germs on everyday objects. To combat these germs and his fear of being dirty, he washes his hands continuously after touching anything with his bare hands. The hand washing has gotten to a point that his hands are red, cracked, and bleeding. What would be some interventions that could help Mikey cope with his OCD? (select all that apply) A. Reassure Mikey that he is not the only one that has these kind of feelings B. Tell Mikey that handwashing will not kill all germs so he still might get sick anyway C. Explain to Mikey that the hand washing is out of control and needs to stop to give his hands a break D. Educate Mikey on how bacteria actually enter the body how they can make patients sick in attempt to reduce his fear of germs

A and D would be the correct answers because A is providing emotional support on dealing with his ineffective coping mechanisms. D would be correct because if Mikey knows how bacteria and "germs" work then he can develop better problem solving skills when it comes to keeping clean in a healthy way. The reference for this reasoning is the NANDA book, nursing diagnosis ineffective coping.

Harold has schizophrenia and is on Haldol. When he shows up for his check up the doctor notices he is walking with a shuffled gait. When the doctor sees him in the exam room Harold's face seems very masklike, and he has a stooped posture. Harold also keeps doing this "pill-rolling" finger movement. What are some medications that would typically be used to help with these symptoms? Select all that apply: A. Artane B. Ativan C. Valium D. Cogentin

A and D. Both are anticholinergic medications which are typically used to treat the pseudoparkinsonism symptoms that Harold is experiencing. Rational for wrong answers: B and C are normally used in patients who are experiencing acute dystonia.

What are some indications that a patient is taking only first generation anti-psychotic medications, rather than only second generation anti-psychotic medications? Choose all that apply. A. Patient c/o feeling depressed B. Patient has normalized serotonin levels C. Patient has no feelings of depression or suicide D. Patient experiencing anhedonia

A and D. This is because first generation anti-psychotics do not work on the negative symptoms of psychosis, some of which are depression and anhedonia. B and C are not correct because second generation antipsychotics help prevent negative symptoms such as depression and they do this by regulating the levels of serotonin in the brain i.e. SSRIs and SNRIs. Source: Davis Drug Guide: SSRI, SNRI, and Haldol

Mary Lou is a 76 yr. old women who has been wandering the unit ever since she got out of her bed 10 minutes ago. You, the nurse, just got done helping your other patients and now, have time to assess Mary Lou. You are able to get Mary Lou back to her bed, but on the way back you ask her what she likes to do for fun and who she likes to have fun with. She responds, "I like drawing and I painted with Picasso last week". When she gets back to her bed, you ask her, "do you know where you are right now?" She says, I'm with my daughter Sara, silly, I'm with you". The next thing Mary Lou says is, "where's your kitchen, Sara?" The nurse is about to utilize a Mini-Mental State Examination (MMSE) along with a functional assessment screening tool (FAST), how should the nurse respond and what tool should she utilize first? Utilize the Mini-Mental State Examination first and tell her, "it's down stairs, are you hungry?" Utilize the Functional Assessment Screening Tool first, and tell her, "it's at my house, not here." Pick which ever one works and tell her, "I'm not Sara, my name is Winnie." none of the above

A because according to (Wilkinson, Barcus, & Wilkinson, 2017), under Nursing Activities and subcategory, "Assessments", "(NIC) Dementia Management, [it first says,] monitor cognitive functioning, using a standardized assessment tool (e.g., the Mini-Mental Status Examination)" and then, to "determine physical, social, and psychological history..." Also, the response, "it's down stairs, are you hungry?" not only acknowledges an unrecognized need, hence the wandering around the unit and asking the nurse, where the kitchen is, but qualifies as a NIC Intervention, subcategory Anxiety Reduction, "minimizing apprehension, dread, foreboding or uneasiness related to an unidentified source of anticipated danger" according to (Wilkinson, Barcus, & Wilkinson, 2017).

Anthony is a 22 year old male who was abused as a child. He never was able to cope with his feelings and felt he doesn't have a strong support system. Anthony made the decision to get help. He mentioned, "My mom used to hit me'' and that he is restless at night. He always feels anxious, gets flashbacks, has nightmares often, and feels very uncomfortable whenever someone touches him. Which nursing diagnosis is best to use for Anthony's situation? A.Post-trauma syndrome related to history of abuse as evidenced by patient states, "My mom used to hit me." B.Social isolation related to history of abuse as evidenced by patient not being able to talk to his friends. C.Disturbed sleep patterns related to physical abuse as evidenced by patient having frequent headaches. D. Acute confusion related to history of abuse as evidenced by patient's frequent headaches.

A is correct because it relates to the situation. Anthony is experiencing post-trauma syndrome based on his statement that his mother hits him. He has classic symptoms of post-trauma syndrome such as flashbacks, restlessness, and nightmares. The post trauma syndrome relate to the history of abuse. Acute confusion is not relevant because the question doesn't show any evidence of confusion or headaches. Lastly, social isolation is not relevant either because it is not certain Anthony has any problems with socialization.

Frank is on the medication clozapine (Clozaril) for schizophrenia. He has been on this medication for three months. Frank comes to the ER with a temperature of 102, a hear rate of 126, blood pressure of 90/60, ulcers in his mouth, and difficulty breathing with O2 saturation of 90%. The doctor has you give him oxygen at 3 liters via nasal canola and his O2 saturation goes up to 97%. What would you recommend the doctor orders next, and why? A. White blood cell count, because Frank possibly has agranulocytosis. B. Red Blood cell count, because Frank is most likely showing signs of Polycythemia. C. An anticholinergic medication, because Frank is experiencing Neuroleptic Malignant Syndrome (NMS) D. An intibiotic, because Frank has a UTI.

A is the correct answer. Agranulocytosis is a possibility when taking Clozaril, and a white blood cell count would be able to show that. All of the symptoms described are typical of agranulocytosis according to the Understanding Pathophysiology book. Rational for wrong answers: B is not correct, because polycythemia is not a possible risk when taking Clozaril, and the symptoms described don't match up with it either. C is no the correct answer, because he is not experiencing NMS, and anticholinergics is not how you treat that anyways. D is not the correct answer, because you try to not order antibiotics unless you know the cause of an infection if possible. Also, if you do more assessments you would probably find that Frank most likely has pneumonia, because that is common with agranulocytosis.

Your patient is on MAOI's for her panic disorder. After talking to her you find out she came into the hospital because of her abnormally high blood pressure. When asking about her diet she voices that avocados is a guilty pleasure of hers and tells you she had one before coming in to see someone about her blood pressure. Why does she have high blood pressure based on the information she told you and what should you educate the patient on? A) Avocados are high in tyramine which has an adverse toxic effect with her MAOI and caused the high blood pressure. B) Educate the patient on the foods that are okay to eat when on MAOI's. C) MAOI's do not cause high blood pressure as a side effect. D) A & B

A&B is correct because MAOI's do have an adverse toxic effect with foods that contain large amounts of tyramine, with avocados being one of them. Also educating the patient on safe foods to eat when on MAOI's. Because many drugs, foods, and beverages can cause an increase in blood pressure in patients taking MAOIs, hypertensive crisis is a constant concern. The hypertensive crisis usually occurs within 15 to 90 minutes of ingestion of the contraindicated substance. Early symptoms include irritability, anxiety, flushing, sweating, and a severe headache. The patient then becomes anxious, restless, and develops a fever.

T.J. has a polygenic disease which she shares with her mother. Her symptoms are rooted from the lack of chemical messages at one point and then an oversupply of chemical messages at another point and receptor insensitivity. What kind of disease(s) could this be? Select all that apply: A. Bi-polar Type 1 B. Depression C. Bi-polar Type 2 D. Schizophrenia

A&C Rationale: Although research suggests that bipolar and schizophrenia disorders may have a similar origins and pathology, Bi-polar disorders are characterized by the lack and/or oversupply of chemical messages to neuro-receptor sites (Page 230). While schizophrenia is believed to be caused by multiple inherited gene abnormalities in combination with non-genetic factors (Page 202). Depression has decreased function of serotonin and nor-epinephrine neurotransmitters, but not an oversupply (Page 253).

You just took over on the day shift and just saw a patient that was presenting tachycardia, sweating, with a very high fever, elevated blood pressure, altered mental state and muscle spasms. After talking to your patient for a while they tell you that they are on a SSRI for depression but wasn't effective so the doctor switched the medication to a MAOI. The patient was supposed to wait 2-5 weeks before starting to take the MAOI but started the MAOI medication only a few days after stopping the SSRI. What do you think the patient is experiencing? Select all the apply. A) Serotonin syndrome. B) Withdrawal from stopping the SSRI medication. C) The patient is experiencing a rare toxic effect of taking SSRI's. D) A&C

A&C The risk of this syndrome seems to be greatest when an SSRI is administered in combination with a second serotonin-enhancing agent, such as a monoamine oxidase inhibitor (MAOI). A patient should discontinue all SSRIs for 2 to 5 weeks before starting an MAOI.One rare and life-threatening event associated with SSRIs is serotonin syndrome. This syndrome is thought to be related to over-activation of the central serotonin receptors caused by either too high a dose or interaction with other drugs. The symptoms are many: abdominal pain, diarrhea, sweating, fever, tachycardia, elevated blood pressure, altered mental state (delirium), myoclonus (muscle spasms), increased motor activity, irritability, hostility, and mood change. Severe manifestations can induce hyperpyrexia (excessively high fever), cardiovascular shock, or death.

N. is a student at Saginaw Valley State University attending the nursing program. Recently, she started feeling "different", she was feeling as though she was Dolly Parton and started singing up and down the halls of HHS. She continually saw rats crawling around Albert E's. She was jumping from the staircase and getting careless for her wound checkoff. A month later she went through a period of hopelessness and having no energy. She couldn't bring herself to get out of bed and shut herself out from the rest of the world. Of the following diagnoses, which are possibilities for P.N? (Select all that apply) A. Bipolar I B. Schizophrenia C. Delirium D. Nothing, these are typical behaviors of nursing students

A, B Rational: Although there is not one specific disease described in this scenario it can be narrowed down to Bipolar I and Schizophrenia. The Foundations of Mental Health Nursing Book says that Bipolar I disorder is characterized by mania and depression. Mania is characterized by the grandiose perception, risky behavior and carelessness. Depression is characterized by the feelings of hopelessness and lack of energy. While the psychosis part is characterized by the visual hallucination.

While taking a patient's vital signs the patient states to the nurse that they have been experiencing insomnia, and are feeling uneasy about a surgery that they have planned. The nurse determines that the patient has moderate anxiety r/t planned surgery as evidenced by insomnia and Pt states "uneasy feelings". What NANDA NOC Outcomes demonstrate an understanding by the nurse of Anxiety Self-Control? (Choose all that apply) A. Coaches patient on how to plan coping strategies for stressful situations. B. Teaches patient how to monitor behavioral manifestations of anxiety. C. Coaches patient to "Relax, nothing ever goes wrong in surgery here." D. Teaches patient relaxation techniques to reduce anxiety.

A, B and D are demonstrations of Anxiety Self-Control outlined in Pearson Nursing Diagnosis Handbook with NIC Interventions & NOC Outcomes (Wilkinson, 2017, pg. 49). Answer C is incorrect because the nurse cannot guarantee that nothing will go wrong.

Jordan is a 10 year old male who recently lost his father in a car accident. Jordan was in the car with his father when he had passed away. The accident was so traumatizing that even now Jordan won't get into a car. He is extremely anxious, his grades have been slipping because he can't concentrate, he doesn't talk to his friends, and he has trouble sleeping. Jordan's mother suggests he goes to seek help. The best nursing interventions for Jordan would be: (select all that apply) A. Teach specific stress management techniques before trauma exploration to restore a sense of control over thoughts and feelings. B. Use developmentally appropriate language to ask about the trauma. C. Pressure Jordan into giving you every detail possible about the accident. D. Focus on self-regulation and rebuilding a sense of security.

A, B, & D are the correct nursing interventions because they take the approach of trauma therapy for a child. Obviously, pressuring Jordan into giving you every detail about the accident is incorrect because all the details are not necessary. The main focus for a nurse is to help Jordan cope and move on with his life. Answer A, B, & D all manifest towards therapeutic techniques.

Anna is a highly educated woman and is a creative film writer. She reports that she spends days at a time writing non-stop and then other days were she can not get out of bed. She has been admitted to the hospital for trying to commit suicide. The nurse goes into do a mental health assessment. What questions are appropriate for the nurse to ask during the assessment? (Select all that apply) A) "Does anyone in your family have mental illness?" B) "Can you tell me more about your non-stop writing?" C) "Is there a stressor that is contributing to your suicidal thoughts?" D) "What are your goals for your treatment here?"

A, B, C Rational: These are the answer to this question because they are ones that would be asked during the assessment. A is a correct answer because in the assessment stage it is important to figure out if there is anything in her history that suggests if she could have a mental illness within her family. Answer B is correct because it is part of an assessment to figure out more about these stages of being awake for long periods of time. C is a correct answer because when looking at someone with potental bipolar disorder figuring out if there is stressor would be important in the assessment stage. D is an incorrect answer because goals are not part of the assessment stage, they are apart of the planning stage (Halter, p224-226, 2018).

Tyler is a 16-year-old who was admitted to a mental health care facility for feelings of suicide. He has been in therapy for 1 year after his twin brother committed suicide. Both brothers have had years of sexual abuse from a 'trusted friend of the family'. Tyler has many risk factors for suicide. What protective factors would decrease Tyler's risk of suicide? SELECT ALL THAT APPLY A) Tyler has effective mental health care and goes to therapy on regular basis. He has regular contact with healthcare professionals. B) Tyler has a strong connection with his family and team mates at school. C) Tyler is good at problem solving. As well as conflict resolution when it comes to his friends. D) Tyler is from a prestigious family that enjoys the finer things in life like exotic vacations.

A, B, C Rational: According to Halter, Varcarolis' Foundations of Psychiatric Mental Health Nursing - 8th edition, Protective Factors are characteristics that make it less likely that individuals will consider, attempt, or die by suicide. These include: (A)• Effective mental healthcare; easy access to a variety of clinical interventions (B)• Strong connections to individuals, family, community, and social institutions (C)• Problem-solving and conflict resolution skills (A)• Contact with providers (e.g., follow-up phone call from healthcare professional) D is not correct because being from a wealthy family does not decrease a person's risk of suicide. However, being impoverished does increase a person's risk.

What assessment findings could lead the nurse to believe that her patient may be experiencing psychosis? (Select all that apply) A)Patient A who mimics the nurse during medication pass. Nurse: "Can you tell me your name and DOB?" Pt: "Can I tell you my name? Can I tell you my birthday?" B)Patient B who states, "I want to go to Florida. I love Florida. Florida makes me happy. My dog makes me happy. Do you have a dog?" in a very hyperverbal manner. C)Patient C whose drug screen came back positive for cocaine and alcohol and states there are children running around his room and he can't get them to leave. D)Patient D who uses a walker to get into the bathroom.

A, B, C Rationale: Persons with psychosis can have the following classic clinical manifestations: Delusions: Fixed beliefs that are not amenable to change in light of conflicting evidence Hallucinations: Perception-like experiences that occur without an external stimulus Disorganized thinking: Most commonly inferred from speech, defined by derailment, loose associations, tangentially, and incoherence Disorganized/abnormal motor behavior: Markedly abnormal behavior ranging from agitation to catatonia that is commonly situationally incongruent Negative symptoms: Alogia, affective blunting, asociality, anhedonia, and avolition

Risk factors for psychosis include: Select all that apply: A)Pre-existing personality disorders B)Child abuse/ adverse child rearing C)An identical twin diagnosed with schizophrenia D)A significant decrease in sleep E)Getting 8 hours of sleep per night

A, B, C, D. Individual risk factors for psychosis include temperament, environmental, genetic, and physiological risk factors (Giddens, 2017). Temperament includes a pre-existing personality disorder that will increase vulnerability to a brief psychotic disorder. Environmental risk factors can include early life experiences in utero, traumatic childhood events, and later life experiences. Genetics plays a role in developing psychosis, with a 48% chance in monozygotic twins, 17% chance among fraternal twins, and 9% chance among siblings. Physiological risks include: decreased cortical grey matter, and a significant decrease in sleep deprivation (Giddens, 2017).

A patient comes to the emergency department complaining of stomachaches, headaches, lethargy, and pain. The nurse recognizes these symptoms can indicate depression, and wants to assess the patient further. Which of the following is NOT a scale that the nurse can use to assess depression? A) Zung Self-Report Inventory B) Scale for Assessment of Negative Symptoms (SANS) C) Beck Inventory D) Patient Health Questionnaire-9 (PHQ-9) E) Geriatric Depression Scale (GDS)

B. The Scale for Assessment of Negative Symptoms is used to screen for schizophrenia (Halter, 2014, pg 123). A, C, D, and E are all scales to identify depression that are completed by the patient.

MK is 68 y/o, has a history of depression and her husband of 35 years just passed away. MK is devastated. She has no interest in eating, cleaning, or doing anything anymore. She is experiencing suicidal ideation and recognizes the need for help. She voluntarily admits herself to the Behavioral Health Unit. The nurse caring for MK recognizes that MK has major depressive disorder and ineffective coping. What interventions could the nurse implement to help MK cope? (select all that apply) A. Encourage verbalization of feelings, perceptions, and fears B. Prepare a quiet, calm area, with ample personal space to facilitate conversation C. Encourage patient to attend religious services D. Provide information about community resources (Support groups) E. Encourage patient to maintain isolation as a method of coping

A, B, D Rational: A lot of patients that are suffering from depression and a loss are angry, so in order for them to cope, they need to verbalize their feelings. Allowing the patient to explore the reasons behind their anger and depression with help facilitate a therapeutic relationship. It is also important to maintain a therapeutic environment when having conversation. Having decreased stimuli will have a calming effect on the patient. It is also important to make the patient aware of coping options once they leave the hospital. Should they start to struggle, support groups can be very effective for talking and sharing feelings.

A patient was just brought into the ER and is experiencing visual hallucinations saying that she is seeing a cat walk around her house when she does not have a cat as a pet. She has been experiencing hallucinations for the past couple of months and then began hearing strange voices telling her what to do and decided to come into the ER to figure out why this is happening to her. What would be a potential cause for these hallucinations that this patient could be experiencing? A. Genes that can be passed from generation that could be coded for psychosis B. Experiencing a traumatic event such as death of a loved one, abuse, or a stressful situation C. Having an allergic reaction to something that you weren't aware of D. Experiencing a traumatic brain injury or another mental condition that affects the brain. E. Abusing medications or other substances that could cause these signs of hallucinations.

A, B, D, E Rationale: All of these choices that are correct when determining a possible factor to why someone maybe being experiencing psychosis. People can begin to start having hallucinations because a gene from previous family members was passed down to them. A traumatic event can also cause them symptoms because it can set someone so much over the edge and start a psychotic episode that they can't control. Depending on what part of the brain is damaged, this person may experience psychosis because of the effect of the accident. The use of medications or other substances can also cause someone to experiences hallucinations and can increase the effect on someone if they are vulnerable to having a psychotic episode.

A patient comes into the ED for what appears to be evidence of self-harm. The patients mother asks what kind of things she should look out for as early warning signs of suicide. (Select all that apply) A. Withdrawing from friends or activities B. Making statements like "I'd be better off dead" C. Hanging out with a new group of friends D. Appearing agitated, anxious, irritable

A, B, D. Rationale: People that are considering suicide sometimes show signs that they are thinking, preparing, or seeking the means to carry it out. C is not correct because this isn't necessarily a bad thing, and could even help someone who has suicidal thoughts.

BC is a patient in your mental health clinical site and has been displaying signs of acute mania. Which of these nursing interventions would be most appropriate in BC's situation? (Choose all that apply) a. frequently remind BC to eat and offer high-calorie, protein drinks, or finger foods b. give simple step-by-step reminders for hygiene and dress c. offer a cup of coffee after dinner to maintain attentiveness d. encourage frequent rest periods during the day

A, B, D. According to Halter (p. 291-292, 2010): (A) constant fluid and calorie replacement is needed, finger foods are good when they can't sit still to eat, and a patient experiencing mania is easily distracted (B) distractibility and poor concentration are countered through simple, concrete instructions (C) avoid giving the patient caffeine after breakfast time to ensure relaxation, rest, and ample sleep (D) lack of sleep can lead to exhaustion and death

A patient who was recently admitted to psychiatric unit with diagnosis of Schizophrenia is currently experiencing positive symptoms. What interventions should the nurse take when assessing for hallucinations? Choose all that apply. A. Assess hallucinations directly. B. Watch for non-Verbal cues. C. Talk to "voices" and tell them to stop. D. Share your own perceptions about voices. E. Assess for anxiety.

A, B, E. Rational: Answer A is correct because all hallucinations must be assessed and monitored carefully for command hallucinations. The voices may command the person to hurt self or others; for example, telling a patient to jump out the window or hurt nursing stuff (Halter, Ch. 12). Asking direct questions such as "Are you hearing voices? or What are you hearing?" gives nurse clues about hallucinations. Answer B is correct because patients may falsely deny hallucinations, requiring observation for behavioral indications of hallucinations such as turning the head as if to listen to someone, suddenly stopping current activity as if interrupted, talking to oneself, moving the lips silently, etc (Halter, Ch. 12). Answer E is correct because in patients with hallucinations intense anxiety can cause not only fear, but symptoms that create further fear. While the person starts to lose touch with reality, their mind is still treating them like they're fine (Giddens, Ch 34 2-nd edition). They may have anxiety because the voices, hallucinations, and loss of social functioning can make it hard to control nervousness.

Cheyanne is a 25 year old female at her PCP's office for a routine physical. Her doctor would like to run a blood test because of her strong family history of breast cancer. Cheyanne has hemophobia, but is extremely concerned about her chances of having the BRCA gene. What are the best nursing interventions you can do for Cheyanne? A. Create an atmosphere to facilitate trust with Cheyanne B. Explain to Cheyanne that the test is important so they can treat her for breast cancer early if she has the BRCA gene C. Provide a safe distraction for Cheyanne to prevent her from fainting D. Tell Cheyanne she probably has the BRCA gene so the test doesn't really need to be done

A, B, and C are the correct answers because they all minimize the stress and anxiety of the situation. A is correct because it builds a rapport with the patient so she does not feel uncomfortable during the blood draw. B is correct because it is important to educate the patient about the benefits of getting the test done. C is correct because it takes the patient's fear into consideration and prevents a fall and anxiety attack from happening. D is not correct because it is giving the patient false information, more anxiety, and ignoring the doctor's orders.

A patient is admitted to the unit with a diagnosis of psychosis. The patient suffers from disorganized thinking, poor hygiene, impaired memory, and anosognosia. Choose the best nursing care(s) to perform that will help treat the patient. Select all that apply: A)Break tasks into smaller, easier to manage steps B)Establish trust and involve the patient in planning of treatment C)Tell the patient that they are dirty and have to be clean to be in the hospitals care D)Concisely and explicitly identify expected hygiene, and provide positive reinforcement for success E)Use restraints because the patient is not safe to be around

A, B, and D. Halter (2018) describes how communication, continuity in care, and trusting relationships with care providers are essential for optimum recovery and relapse prevention. The patient is suffering from poor hygiene, disorganized thinking, and impaired memory. The best nursing care for a person with these psychosis symptoms would be to 1) concisely and explicitly identify expected hygiene, while providing positive reinforcement for success, 2) break tasks into smaller, easier to manage steps, and 3) use visual cues to prompt attention to hygiene tasks (Halter, 2018, p. 205). The patient is also experiencing anosognosia r/t their psychosis. Halter (2018) also describes that the nurse should 1) not be judgemental and treat the patient with respect, 2) establish trust and involve the patient collaboratively in planning treatment, and 3) explain in a clear, concrete, and confident manner your belief that the patient will benefit from treatment.

Chris has been feeling very lethargic lately; not much of anything is inspiring him to get out of bed, and he feels as if he has nothing to look forward to or to keep him happy each day. His mother is struggling to help her son regain some energy and happiness in his life as he refuses to do things with her and feels as if nothing he does is good enough. A couple weeks later, he voluntarily admits himself into a mental health unit to help him with his depression. Which of the following Nursing Interventions would be appropriate for Chris whom is feeling hopeless. Select all that apply. A. Emotional support - giving him reassurance and acceptance. B. Support group - allowing him to be in a safe environment to provide support and understanding with others whom may be suffering C. Telling his mother his personal stories that he has opened up about upon admission to the unit behind his back D. Assuring that he does not talk about the things in the past that has made him feel sad and depressed, and simply focus on the future because that's all that matters E. Being there for Chris, and making sure that he knows he is in a safe environment and has someone to come to in times of need

A, B, and E - these would be the correct answers for this question based on the NIC Interventions within the NANDA book. The book states that emotional support, a support group, and the presence of someone are all appropriate nursing interventions for someone feeling hopeless (Wilkinson, 2017, 418-419). Telling his mother information about stories he has opened up about will break his trust and would not be appropriate, meanwhile encouraging him to disregard his past feelings and not talking about what had triggered these new feelings in the first place would not be the best intervention for this patient either.

JJ is in the hospital after his roommates brought him in concerned about his depression. JJ is a chef at an asian restaurant and usually will eat whatever he cooks up that day. He has been taking his meds regularly and only recently has his roommates noticed a change in his appearance and the way he acts. The nurse looks over his charts and sees that he has been on an antidepressant called Nardil and realizes this is an MAOI. Her next action would be to question what types of foods he is eating to see if this medication is being affected in a negative way. What foods should JJ not being eating Select all that apply A. Aged Cheeses B. Salami C. Fresh Fruit D. Fresh Vegetables E. Salty sauces (soy sauce, fish sauce)

A, B, and E. The use of MAOIs has been rethought because of all the possible effects.An enzyme called monoamine oxidase is involved in removing the neurotransmitters norepinephrine, serotonin and dopamine from the brain. A MAOI blocks this to keep the depression down. The foods he is/was eating is a concern because it works against the drug and will cause it not to work appropriately. (Halter, 2014.)

If your patient is displaying the following: lack of energy (anergia), lack of motivation (avolition), and is unable to initiate tasks (social contact, grooming, and other aspects of daily living), what nursing diagnosis' would you find in their care plan? (Select all that apply) A. Ineffective coping B. Self-care deficit (bathing/hygiene, dressing/grooming) C. Impaired verbal communication D. Risk for self-directed violence

A, B. Rationale: This is correct because the patient is displaying signs of lack of energy and lack of motivation, which goes with ineffective coping. If they are unable to initiate tasks, they are unable to take care of themselves. There is nothing about verbal communication within the question And the information does not say that the patient is suicidal.

Which of the following statements would NOT demonstrate good examples of a mentally healthy individuals resilience? (Select all that apply) A. "I tend to keep to myself. When I talk to others, I usually just end up acting out due to my emotions." B. "Everyday I wake up with the mindset of making today a great day." C. "Most days, my emotions are so strong that it impairs my work, school, and personal life." D. "When I face an obstacle, I find that it really helps me to talk to others about it."

A, C Rationale: Resilience is defined in the book as "the ability and capacity for people to secure the resources they need to support their well-being". A mentally healthy individual has the ability to be optimistic, has a sense of mastery, and feels competent. Resiliency is a major factor in the recovery of a traumatic event. Answers A and C both show an individual who does not feel in control of their emotions. Answers B and D show an individual who feels like they are in control of their emotions, feel competent to talk about their feelings, and handle obstacles. This question is asking for the statements that do not show a good example of resiliency, so A and C would be the correct answers.

Jim is alert and oriented to person, place, and time. He feels he cannot leave for work unless he has reorganized his shoes five times and checked that his front door is locked five times. This behavior makes him late every day. What is the best NANDA for this patient's behavior? A. Ineffective coping r/t inability to deal with changes associated with aging B. Ineffective coping r/t unrelenting repetitive thoughts to perform irrational activities C. Acute confusion r/t sensory overload D. Ineffective role performance r/t impaired social interactions

B because he is exhibiting symptoms of OCD. He does not seem to have trouble with his age. He is not acutely confused because these behaviors are more obsessive compulsive than confused. He is A&O x3 so his cognitive function is intact, but his compulsive behaviors are making him late to work. He may have ineffective role performance at work because of this, but this is a result of his compulsive behavior and is not the underlying nursing diagnosis. References from the NANDA book.

Many neuroimaging techniques can be used to better visualize the innerworkings of the brain, some of these imaging techniques include computed tomography (CT), magnetic resonance imaging (MRI), and positron emission tomography (PET). When evaluating these imaging studies for a patient with a history of bipolar disorder, you would expect to see dysfunction in what region(s) of the brain? (Select all that apply) A: Prefrontal cortical region B: Hypothalamus C: Hippocampus D: Amygdala

A, C, D According to Halter Chapter 13, someone who has bipolar disorder will have dysfunction in the area of the brain associated with "executive decision making, personality expression, and social behavior" (pg. 222), this is the prefrontal cortical region. The hippocampus which is associated with "memory and the amygdala which is associated with memory decision making and emotion. The dysregulation of these areas will result in the lead to the signs of emotion lability, heightened reward sensitivity, and emotional dysregulation" associated with bipolar disorder. Answer B (hypothalamus) is incorrect because the hypothalamus is associated with coordinating the autonomic nervous system and the pituitary gland which are not directly related to the signs and symptoms associated with bipolar disorder.

Jason Smith is a 34-year-old man suffering from symptoms of psychosis. Doctors have been working on adjusting his medications due to a recent encounter with a family member. He tells the night nurses "I slapped my brother when he saw the dog running across the bridge to the shelves" when asked about the event. The nurses realize the patient is displaying signs of associative looseness. The correct and most therapeutic way(s) for the nurse to respond to the situation include(s): (Select all that apply) A. "I am having trouble understanding what you are saying, Jason." B. "I understand what you are trying to say. We will take care of it." C. "You've mentioned trouble with your brother before. Tell me more about the last time you talked with him." D. "I do understand from previous conversations your brother is important to you. Correct me if I am wrong."

A, C, D Rationale: Patients dealing with psychotic like symptoms often display associative looseness, or extreme disorganized thinking. When dealing with patients, (A) is correct because it is important to place the difficulty in understanding on yourself as the nurse, not the patient. As a nurse, one should NOT pretend or allow the patient to think that you understand when you don't, therefore, making (B) false. The letter (C) is correct because Halter mentions it is important to look for reoccurring themes with the patient's communication that could potentially be possible triggers. The letter (D) is also correct due to the nurse's ability to summarize the patient's communications with the chance for the patient to correct anything one may have misunderstood.

Joan is a patient on the Mental Health Unit. She has a history of psychosis, but her symptoms have been under control lately with the help of her medications. She is currently exhibiting abnormal behavior and was seen talking to herself and mumbling under her breath. She seemed to be getting angry with someone although she is alone in her room. Which of the following would be considered therapeutic communication for Joan? (Select all that apply). A. "What are you hearing?" B. "You don't hear that. There are no voices." C. "The voice you hear is part of your illness. It cannot hurt you." D. "Try to listen to me and others that you can see around you."

A, C, D Rationale: B would not be considered therapeutic communication because you should not negate the patient's experience. It is important to tell the patient what they are experiencing is not real, but there are more therapeutic ways of doing so. For example, it's okay to say "I do not hear those voices, but that must be scary for you".

Mr. Thomas is a 76 year old male whose daughter brought him into clinic for evaluation. His daughter states that her father has been spending more and more time sleeping, has withdrawn from his regular activities, has had a decreased appetite, and has "seemed sad and not himself." In reviewing the patient's EMR, the nurse learns that he has several co-morbidities. Which of the following co-morbidities does the nurse know can mimic the signs and symptoms of depression? (Select all that apply) A) Hypothyroidism B) Diabetes mellitus C) Congestive heart failure D) Sleep apnea E) Rheumatoid arthritis

A, C, D, and E. According to Halter, hypothyroidism can result in the appearance of depression, while hyperthyroidism can appear to be a manic episode (Halter, 2014, pg 120). Congestive heart failure and sleep apnea can both cause hypoxia, which can manifest as the symptoms of depression (Halter, 2014, 119). It is believed that the chronic inflammation caused by rheumatoid arthritis is the cause of depression symptoms in some sufferers. B is not correct because while Diabetes mellitus can cause mood changes due to rising and falling blood glucose levels, these changes are quick in onset and departure, differentiating from depression.

Lisa is a 24 year old who was diagnosed with major depression after losing her mother two years ago. She has previous attempts of suicide and has admitted herself into the mental health unit after another attempt two days ago. She states "I just this to all be over and will do anything to help me stop feeling this way." Which nursing interventions would be most appropriate for this patient? (Select all that apply) A. Educate the patient about risks, warning signs, and prevention factors for suicide B. Move her into seclusion until she starts to feel better C. Provide her with information for a crisis counselor D. Inform her on social support groups she could attend

A, C, and D because according to Halter (2014), interventions for suicide patients include primary, secondary, and tertiary interventions. Primary interventions would include education, support, and information to help prevent suicide. Secondary interventions are to treat the suicide crisis by counseling sessions. Tertiary interventions are interventions that include other people who have attempted suicide like a support group. Putting the patient into seclusion is not going to help in this situation but could actually make it worse. (487- 491)

J.J. Is a 47 year old male, who is married and has 5 children. He is at his primary care physician's office because he has been having lower back pain for 8 months now and doesn't know what to do about it. He also wants to discuss with his doctor about feeling "down" more often than not lately. What are some things about J.J. that put him at a higher risk for committing suicide? Choose all that apply. A. He is 47 years old B. He is married with 5 children C. His chronic lower back pain D. He is male

A, C, and D. A. According to Halter, for men suicide rates peak after the age of 45. C. In Halter it states that "About half of those who complete suicide have physical illnesses. Loss of mobility, disfigurement and chronic pain are especially associated with suicide." D. Halter also states that "Men committ suicide four times more often than women." B. is wrong because in the Halter textbook it states that being married with children in the home significantly reduces the risk of suicide. The above question information was referenced from "Foundations of Mental Health Nursing, Halter, 7th edition, page 482

You arrive for night shift and learn that you are the nurse assigned to a "difficult" patient on a medical/surgical unit. The nurse giving you report states, "Patient M.L. is a 82 year old woman being treated for a severe case of the flu. She was cooperative, calm, and resting quietly before I gave her 1600 medications. Around 1800, she refused to let the CNA change her bedding after she wet herself, pulled out her IV line, and demanded that her husband leave the room and get her strawberry ice cream. Her husband passed away last year. We can't get calm her down." Which of the following are appropriate questions to ask the nurse giving report? Select all that apply. A. "Have you checked her electrolyte levels?" B. "Did you try to restrain and sedate her?" C. "Have you completed a CAM assessment?" D. "Does she have a hx of mental illness?"

A, C, and D. According to Giddens (2017), delirium is a state of disturbed consciousness and altered cognition with a rapid onset, and that it is the most frequent complication of hospitalization in the elderly population. A is a good question to ask because unbalanced electrolyte levels are a potential cause for delirium. B is a good question to ask, but not the most appropriate here, as the nurse is trying to assess for delirium. C is a good question to ask because the CAM assessment is used to assess delirium. D is a good question to ask to compare the patient's baseline to what her current behaviors are.

MT is a patient with a history of depression that results a lot of negative self-perception such as self-blame and discounting of positive attributes. What are some effective coping mechanisms that you could suggest to the patient? (Choose all that apply). A) Encourage formation of supportive relationships B) Encourage them to keep all of their emotions inside of them C) Encourage exercise such as running or weight-lifting D) Encourage them to ignore their feelings and pretend like they aren't effected by them

A, C. In the Halter (2018) book table 14.4 offers guidelines for counseling people with depression. According to the table, supportive relationships can "help reduce social isolation and enable the patient to work on personal goals" and exercise can help improve self-concept by "improving the brain's neurochemistry" (Halter, 2018, pg. 255).

There are many risk factors considered when assessing a patients risk for developing a bipolar disorder. Select all of the risk factors below that ARE related to developing a bipolar disorder. A: the illness runs in the family B an imbalance of neurotransmitters and receptor site insensitivity C: hypothyroidism D: recent high school graduate applying for college E: a spouse is diagnosed with bipolar disorder

A,B,C Rationale: There is no evidence suggesting that recently graduated high school students, or having a spouse with bipolar disease increases the risk for an individual to develop a bipolar disorder. Halter, Chap. 13, pg. 225-26, states that bipolar disorders tend to run in families, and that the lifetime risk for individuals with an affected parent is 15-30% higher. Halter also states that in simple terms too few of neurotransmitters can result in depression, too many can result in mania The sensitivity of receptor sites can also be the root of the problem if the neurotransmitters cannot be received where they are supposed to go. Hypothyroidism is one of the most common physical abnormalities associated with bipolar disorder.

Jack is your patient that was newly admitted to the ER as a result of being cut with a knife during a fight. He willingly tells you he was diagnosed with Bipolar disorder a couple of years ago and has recently started self medicating with alcohol again. You cannot get him to sit still, he is very confused as to how he got cut, and he appears to be underweight. As his nurse, what nursing actions should you take in this situation (choose all that apply)? A. Offer Jack water to keep him hydrated B. Calmly explain the situation to Jack and make sure he understands C. Ask the provider if he thinks it would be a good idea to sedate Jack D. Keep asking Jack questions to try and get him to remember

A,B,C A: It is important in the manic state that the patient remains hydrated as it is common for them to be constantly moving and may not have eaten or drank anything for a couple of days (Halter, 2018, p. 229). B: During the manic phase it is common for the patient to have confusion related to sleep deprivation because they often don't feel the need to sleep for days at a time (Halter, 2018, p. 230). C: As a nurse, it is your job to intervene when you feel it is necessary to help a patient and do what is best for them. Simply suggesting the doctor should consider sedating Jack is only in his best interest since it is a probability his confusion and agitation are related to sleep deprivation (Halter, 2018, p. 230). D: Asking Jack further questions at this particular time would only confuse him more and have the potential effect of aggravating him even more. Instead, redirecting his attention to a T.V. or magazine would be more effective in making sure his mania is better controlled and his frustration is diminished (Halter, 2018, p. 231)

What are some of the underlying conditions that can lead to someone experiencing delirium? Select all that apply. A. Dehydration B. Electrolyte imbalances C. Adverse effects of medication D. Illicit drug use E. Fever

A,B,C,D,E. According to Giddens (2017), all of these are underlying conditions that can cause a patient to experience delirium. (p. 339)

Lisa, an 18 year old senior in high school, noticed her best friend was acting out. She had started fighting with classmates, disrupting class and even partaking in sexual encounters/drug use. This behavior was unlike her, Lisa later learned that her friend had been diagnosed with Bipolar disorder and wondered what her own chances were of having it. Which of these factors could put Lisa at greatest risk A. Losing a family member B. Having a parent with Bipolar disorder C. Having an aunt with Bipolar Disorder D. Hyperthyroidism

A,B,D According to Halter CH 13, Risk factors for Bipolar disorder include Genetics, neurotransmitters, neuroendocrine, Environmental Stressors and physiological factors. Stressful family life and adverse life events such as losing a job, family member, divorce or even abuse can trigger mania or depressive episodes. Bipolar disorder runs in families, and the lifetime risk for individuals with an affected parent is 15% to 30% greater. The thyroid is important for producing hormones in your body that affect the body's energy. When there is an increase, a person could be in a manic state and when there is a decrease they can experience depression. C would not be a correct answer for this question because an aunt is not a first level relative, therefore not having the greatest affect on the individual and is therefore not the best answer choice.

RL has chronic kidney disease and is getting discharged from the Behavioral Health Center. His physician puts him on divalproex sodium (Depakote) to treat his Bipolar Disorder. The nurse notices that divalproex sodium was prescribed instead of Lithium because (Choose all that apply): a. Lithium has a small window between therapeutic and toxic levels b. Lithium has a contraindication with renal disease c. divalproex sodium requires no lab testing d. Lithium has many unwanted side effects

A,B,D According to Halter Ch 13, pg 233-234: Newer drugs have resulted in a decline in lithium use. Newer drugs have been introduced and approved to carry lower toxicity, have more favorable side effects, and require less frequent laboratory testing. A. There is a small window between the therapeutic and toxic levels of lithium. Lithium must reach therapeutic blood levels to be effective. This usually takes 7 to 14 days, or longer for some patients. Blood serum should reach a level of 0.6 to 1.2 mEq/L (Sadock et al., 2015 (Links to an external site.)Links to an external site.). Lithium levels should not exceed 1.5 mEq/L to avoid serious toxicity. B. Lithium therapy is generally contraindicated in patients with cardiovascular disease, brain damage, renal disease, thyroid disease, or myasthenia gravis. Whenever possible, lithium is not given to women who are pregnant because it may harm the fetus. Lithium use is also contraindicated in mothers who are breast-feeding and in children younger than 12 years of age. C. Depakote does require some blood draws. Blood draws may be done once or twice a week but once symptoms are controlled, the draws become less frequent. Lithium requires frequent lab draws. Lithium levels should be measured at least 5 days after beginning lithium therapy and after any dosage change, until the therapeutic level has been reached. Blood levels are determined every month. After 6 months to a year of stability, it is common to measure blood levels every 3 months. For some people, the demand of blood draws may not work with their lifestyle. D. Expected Side Effects of Lithium include: Nausea, vomiting, diarrhea, thirst, polyuria (producing too much urine), lethargy, sedation, and fine hand tremor, Renal toxicity may occur with long-term use, Goiter and hypothyroidism. Lithium has less favorable side effects than newer drugs.

A nursing student has just finished listening to a lecture about Major Depressive Disorder. This nursing student is reviewing her notes with a friend after class and sharing what she has learned. Which of the following statements shows the nursing student DID NOT understand the information presented in the lecture: (select all that apply) A) "Depression effects mostly adults so it isn't really important to screen for depression in adolescents" B) "Depression can effect any age range from the elderly to infants" C) "Depression in children isn't really common" D) "Screening for depression in any age patient is important"

A,C Reasoning: "Because symptoms vary by age and circumstance, depression in children, until recently has been underrecognized. We now know that even infants can display symptoms of depression (Jacobs & Taylor, 2009)." (Halter, 2014, pg. 251) Depression is not age specific. It is important for all nurses to screen for depression any age of patient if it seems warranted.

Recently self-admitted the behavioral health department, Abe a 40-year-old male reveals that he has been excessively worried about his marriage, his job security and his finances. Abe states that he has felt very irritable and has not been sleeping well. Some nights he says he doesn't sleep at all. Abe states that he can't shake the feeling that something bad is going to happen. Since his admission Abe was diagnosed with unspecified severe anxiety. What medication administered to Abe demonstrates the nurses understanding of treatment for his diagnosis? A. Duloxetine (Cymbalta) B. Lithium (Carbolith) C. Quetiapine (Seroquel XR) D. Benztropine (Cogentin)

A. A. Indications: Generalized anxiety disorder. MOA: Inhibits serotonin and norepinephrine reuptake in the CNS. Both antidepressant and pain inhibition are centrally mediated. Therapeutic Effect(s): Decreased symptoms of anxiety. B. Indications: Manic episodes of bipolar I disorder (treatment, maintenance, prophylaxis). C. Indications: Schizophrenia, Depressive episodes with bipolar disorder, Acute manic episodes associated with bipolar I disorder (as monotherapy [for adults or adolescents] or with lithium or divalproex [adults only]), Maintenance treatment of bipolar I disorder (with lithium or divalproex), Adjunctive treatment of depression. D. Indications: Adjunctive treatment of all forms of Parkinson's disease, including drug-induced extrapyramidal effects and acute dystonic reactions.

Max is a 19 year old white male that was brought into the ED. While in the being interviewed Max was very quiet,did not talk much, and avoided eye contact. During the interview he stated, "There is no meaning to life." Max also revealed that he failed out of college and broke up with his girlfriend. He also stated that he felt helpless and hopeless. What is the priority NANDA for this patient? A. Risk for suicide, related to age, gender, helplessness and hopelessness. B. Risk for self-mutilation, related low-self esteem and adolescence. C. Ineffective impulse control, related to hopelessness. D. Ineffective coping, related to insufficient sense of control.

A. Answer A is the best because of the patient's situation. Adolescents and younger adult males are at a higher risk for committing suicide than others. The patient also stated that he felt helpless and hopeless. These are two risk factors for committing suicide. The patient's statement that he felt like there was no meaning to life shows that he is ready to die and puts him at a higher risk for committing suicide. The patient also feels lonely since he failed out of college and he broke up with his girlfriend. This is also a risk factor for suicide.

K.K. is a 17 year old female recovering from a broken leg after a car accident. The doctor was just in and told K.K. that she will likely not be able to participate in the state track meet in a couple of months. The nurse comes in and checks on K.K. K.K.'s recent vitals were BP 125/70, P 72, RR 16. Her pulse is now 100 and her respirations are now 20. The nurse also notices that she is rocking her hand back and forth. The nurse asks K.K. how she is feeling and the notices tremors in her voice when she starts to explain the doctor's news. K.K. focuses on not being able to participate in the track meet but can carry a conversation with the nurse. What is the most appropriate intervention for K.K.? A) Encourage K.K. to talk about her concerns B) Administer medication for a panic attack C) Move K.K. into a quieter setting D) Quickly change the topic to a more positive conversation

A. On page 271 of Varcarolis' Mental Health Nursing book, it gives the signs and symptoms of different levels of anxiety. K.K. is displaying signs of moderate anxiety. In moderate anxiety, patient's can engage in tension relieving behavior such as pacing or rocking their hand back and forth. They can have increased respirations and pulse. K.K. can explain what the doctor told her, so that is why she is not experiencing severe anxiety. On page 285, there is a table of interventions for mild to moderate anxiety. It is important to encourage the patient to talk about his or her feelings or concertns because this allows for the patient's feelings of isolation to be decreased. If the topic of communication is changed quickly, it does not allow the patient to fully cope and allows them to feel isolated. The interventions for severe to panic levels of anxiety are found on 286. Interventions for these include moving the patient into a quieter setting to minimize escalation. K.K. is still able to function and problem solve, so this intervention is not needed. Medication administration is not the first intervention without full assessment or order for mental health patients.

J.D. has been admitted to the hospital for psychosis. After taking some patient history, the nurse notes that he has been taking first-generation antipsychotics for a few months. The nurse has been walking the halls to check on her patients when she notices J.D. is pacing his room and will not sit down or be still. What is the appropriate assessment and/or intervention the nurse should recognize? a. The patient is noncompliant and put him in restraints b. The patient is experiencing side effects from their medications and the nurse should talk to the physician about lowering the dose or switching it. c. The patient is feeling anxious and may need another dose of their medication. d. The nurse should recommend to the physician that he should take a look at the patient about anxiety.

B According to the Halter book, first-generation antipsychotics have a generally higher level of side effects. One of the side effects is akathisia, which is defined as, "motor restlessness that causes pacing, repetitive movements, or an inability to stay still or remain in one place" (Halter, 2017, p. 211). Restraints should be the last resort; it should only be used if the patient is going to harm someone or them-self. It also says akathisia can be mistaken for anxiety or agitation, but again that should not be the first resort because if more medication is given it could make the side effect worse, which could turn into tardive dyskinesia. This is why B is the correct answer because we know FGAs come with a higher risk of side effects and if we know what those side effects look like then we can try and stop it.

According to the DSM-5, in order to be diagnosed with schizophrenia, patients must present with two or more specified symptoms, each present for a significant portion of a one-month period (or less if undergoing treatment). Which of the following symptoms are included on the diagnostic criteria for schizophrenia? (Select all that apply). A. Suicidal thoughts B. Hallucinations C. Obsessions D. Delusions E. Disorganized speech

B, D, & E according to the "DSM-5 Criteria for Schizophrenia" chart on page 192 in the Halter textbook published in 2017. According to Halter (2017), all patients diagnosed with schizophrenia have at least one psychotic symptom such as hallucinations, delusions, and/or disorganized speech or thoughts (pg. 192) This rational explains why the correct answers are B, D, & E. C is incorrect because obsessions are a characteristic of obsessive compulsive disorder rather than schizophrenia. A is incorrect because suicidal thoughts are seen more often with patients who have a decreased mood paired with high energy levels.

T.D is a 28-year-old male that has been using crystal meth for many years. He continually does it with his girlfriend because he likes the way it makes him feel. After using this multiple times, he began to feel his heart racing and didn't know why this was. He ignored these symptoms and continued to use meth like he always did. The racing heart did not go away and began to hear voices that keep him up at night. His girlfriend was not experiencing these and did not understand what he meant. He decided he needed to talk to someone about this. His friend is a nurse and he asked for her advice. His friend told him that he could be experiencing psychotics symptoms from the drug. What should the nurse tell him about the effects of methamphetamines and psychosis? A. "The more you use crystal meth, the quicker the symptoms will go away" B. "If you stop using crystal meth, you will have a higher recovery from these symptoms once the drug is out of your system" C. "The symptoms will not go away because you have been on the drug for so long" D. "I am a nurse so I cannot give you information about this"

B According to Giddens, a lot of reasons why someone experiences psychosis is because of drug induced reasons. Symptoms with drug induced psychosis can be acute and will usually go away once the drug is out of the person's system.

A patient was admitted to the unit after attempting suicide by over dose. After the patient arrives, a nurse over hears the patient saying "Being here isn't going to do me any good, I have nothing to live for anymore and no one who cares about me. This world is better off without me" Which nursing diagnosis best fits this patient? A. Social Isolation r/t fear of rejection B. Risk for self harm r/t feelings of hopelessness and loneliness C. Low self esteem r/t feelings of shame and guilt D. Self care deficit r/t anergia (Lack of motivation) E. Anxiety r/t poor interpersonal relationship

B Rational: While some other diagnosis could work for this patient, the BEST choice is Risk for self harm. According to NANDA the biggest risk factors for this diagnosis are a hopeless demeanor, which this patient demonstrates by statements of "I have nothing to live for anymore". Also, History of attempted suicide, which this patient also has. And lastly, NANDA shows that certain verbal clues are a big indicator of this diagnosis. In this scenario patient statement of, "this world is better off without me" indicate a risk for further self harm.

The family of a patient who attempted suicide is wanting to learn more ways that they would be able to care for their father upon having him discharged from the hospital. If the family is very supportive and wanting to help what information should the nurse be sure to include for them? A. Only address the situation if a note is formed and how the plan was meant to be carried out B. Be able to listen to what is actively going on, offer support, and accept how their father is feeling C. Seclude him off in an environment away from things that he could use to harm himself D.Try to distract him by doing activities at the beginning of every hour

B Rationale: The most important of the information that the nurse should include for them is to be able to listen to what is actively going on, offers support, and has to be able to accept how their father is feeling The other ones are not ways to be supportive towards your father. According to the National Suicide Prevention Lifeline, how to help a person is to ask and listen, be understanding, give a hug, and get them help and take care of yourself (National Suicide Prevention Lifeline)

James is a 23 year old male and is diagnosed with major depressive disorder. Which nursing diagnosis should a nurse assign to this client to address a behavioral symptom of this disorder? A. Altered communication R/T feelings of worthlessness AEB anhedonia B. Social isolation R/T poor self-esteem AEB secluding self in room C. Altered thought processes R/T hopelessness AEB persecutory delusions D. Altered nutrition: less than body requirements R/T high anxiety AEB anorexia

B A nursing diagnosis of social isolation R/T poor self-esteem AEB secluding self in room addresses a behavioral symptom of major depressive disorder. Other behavioral symptoms include psychomotor retardation, virtually nonexistent communication, maintaining a fetal position, and no personal hygiene and/or grooming (NIMH, 2016).

A client diagnosed with schizophrenia tells a nurse, "The "Shaperonians" took my shoes and purse out of my room last night after I fell asleep." Which is an appropriate charting entry to describe this client's statement? A. "The client is experiencing command hallucinations." B. "The client is expressing a neologism." C. "The client is experiencing a paranoid delusion." D. "The client is verbalizing a word salad."

B Rational: The nurse should describe the client's statement as experiencing a neologism. A neologism is when a client invents a new word that is meaningless to others but may have symbolic meaning to the client (according to Halter book, Ch. 12). Command hallucinations are auditory hallucinations that instruct a patient to act in specific way. Paranoid delusions are false belief that one is being harmed or persecuted by a particular person or group of people. Word salad refers to a group of words that are put together randomly.

James has been admitted to an inpatient psychiatric unit via petition and certification. What is the rationale for a nurse to perform a full physical health assessment on James, who has a diagnosis of major depressive disorder? A. The attention during the assessment is beneficial in decreasing social isolation. B. Depression can generate somatic symptoms that can mask actual physical disorders. C. Physical health complications are likely to arise from antidepressant therapy. D. Depressed clients avoid addressing physical health and ignore medical problems.

B The nurse should determine that a client with a diagnosis of major depressive disorder needs a full physical health assessment because depression can generate somatic symptoms that can mask actual physical disorders. Somatization is the process by which psychological needs are expressed in the form of physical symptoms (NIMH, 2016)

Jennifer, 24, went to the doctor complaining that she hasn't been feeling herself for the last few months. After doing an assessment, the doctor diagnosed her with Bipolar Disorder. What signs and symptoms were likely present in order to come to this diagnosis? A. Depressive and manic episodes frequently throughout a day caused by random triggers B. Depressive state lasting at least 2 weeks and persistent manic state lasting at least 4 days C. Depressive episode one day and manic the next D. Manic state only

B According to Giddens pg. 320, everyone will experience episodes of sadness, irritability, and even euphoria or rage occasionally however, the persistent and frequent episodes show instability. A and C are not the right answers because they do not show consistency. The individuals mood changes frequently which is not a sign of bipolar disorder. D is also not a correct answer because Bipolar disorder consists of episodes of both manic and depression.

Robert comes into the ED with a history of diabetes, HTN, bipolar disorder, and hypothyroidism. He is A&Ox3, HR: 105 regular, BP: 106/70, RR: 14 regular. He is experiencing N&V, diarrhea, blurred vision, polyuria, and polydipsia. When questioned about what medications he is on he states "I take a sugar pill for my diabetes and a pill to control my mood swings, but I can't remember their names". Based off of the presenting signs and symptoms the most appropriate nursing interventions would be: (Select all that apply) A. Obtain a blood glucose level because the pt is in a diabetic crisis B. Assess renal function to rule out toxicity due to his medications C. Monitor the pt's thyroid levels D. Carry out contact precautions when assessing this pt because he has influenza

B & C It can be inferred that the med the pt is taking to control his mood swings is lithium. This is because he presents with the classic flu-like symptoms of lithium toxicity. It is important to monitor thyroid function because lithium toxicity can cause hypothyroidism and the pt already has low thyroid levels. Renal toxicity is a common side effect, especially if the pt is receiving too high of a dose (Halter, 234). Even though the pt has diabetes, he is A&Ox3 and his respirations are regular and within normal range so he is not in a diabetic crisis. It would be a good idea to obtain a glucose level, however it is not a priority intervention in this situation.

Depression is a common mental illness that affects around thirteen million individuals annually in the United States. The common signs/symptoms of depression are anhedonia, fatigue, sleep disturbances, irritability, feelings of hopelessness, etc. Depression can vary depending on the age and circumstances of the individual. Because of this it has gone underrecognized in certain populations. What below populations are underrecognized for depression? A) Elderly (65+) B) Infants C) Children D) Adults E) All the Above

B & C (Halter Pg 251). Children and Infants depression tends to go unrecognized because the signs/symptoms do not look like your common symptoms. In these populations the signs/symptoms tend to be more irritability. In elderly and adults you would normally see the common signs/symptoms. Because of this and their ability to express what they are feelin and going through they do not go underrecognized. According to Halter "Children tend to be irritable rather than depressed. It may be a single episode or recurrent (more than one) episode." Also, according to Halter "Because symptoms vary by age and circumstance, depression in children, until recently, has been underrecognized. We now know that even infants can display symptoms of depression" (Jacobs & Taylor, 2009).

Which of these patients are in a vegetative state based on their signs and symptoms and level of awareness? (Select all that apply) A) Doug with a brain stem hernia who doesn't respond to voice or light pain. B) Chelsea with reflexes to light pain and a startle reflex to voice. C) Mike with no awareness, but can yawn, sneeze, and swallow. D) Rachel with some awareness and purposeful actions.

B and C. Giddens (2018) states that when one is in a vegetative state, there is no awareness or purposeful behavior. Patients can yawn, sneeze, and swallow, and have reflexes to light pain. Chelsea and Mike are the only patients with this condition.

A 25 year old female has presided in the ICU/CCU for the past four months due to a MVA. Upon shift change, The patient's oncoming Nurse notices that she yawns every now and again. The Nurse pinches the patient's great toe, and the patient pulls away slightly. The Nurse notes that the patient will open her eyes only for a half hour or so, and then drift back off into a sleep. The nurse understands that This patient is fluctuating between which types of conscious states? (Select all that apply) A. Comatose State B. Vegetative State C. A&O times 3 D. Minimally Conscious State

B and D, because according to the level of conscious summary by B. Harmer, a person in the vegetative state usually does not speak unless startled, but it is only a reflex. Also, the patient only has a reflex to light pain. After someone comes out of a coma, they may drift back and forth from a vegetative state to a minimally conscious state. The brain-stem, thalamus, and pons are still intact, so the patient may open their eyes, sneeze, yawn, or twitch, but again these are just reflexes (Harmer, 2018).

While doing your morning medication administration, you observe a patient named Amy, a 22 year old female experiencing Psychosis, avoiding interaction with other patients. What nursing interventions would be appropriate to include in her care? (Select all that apply) A. Explain to Amy how important social interaction is in order to be healthy. B. Ask Amy to join in on scheduled activities that are already going on throughout the day C. Approach the patient and take the time to talk with them for a long period of time each day. D. Assure Amy that she has the power to choose how long she has to partake in activities involving social interaction.

B and D. According to Halter (2017), encouraging the patient to partake in group activities is a recommended method to encourage the patient to interact without feeling pressured. By assuring Amy that she can come and go when she wants to, it allows her to feel empowered to make her own choices, and therefore feel more accepted and willing to interact. Rationale: The answer of A would be incorrect because it does not "Convey acceptance and meet the patient where they are now" (Halter, 2017, pg. 206). On the contrary, it does the opposite of the intention, and makes the patient feel like they are doing something wrong. The answer of C would be incorrect because it would make the patient feel uncomfortable and retreat further into their isolation. Instead of engaging right away with long discussions, you should instead "Connect at intervals and interact briefly about low anxiety topics" (Halter, 2017, pg. 206).

A mental health patient has all of these rights to treatment EXCEPT (select all that apply): A.) The right to privacy and dignity B.) The right to a private room C.) The right to be free from excessive or unnecessary medication D.) The right to the least restrictive environment E.) The right to leave treatment at any time

B and E Rationale: A.) is not the correct answer because all patients do have the right to privacy and dignity. B.) is the correct answer because patients do not have the right to a private room, but may receive a private room based on an evaluation or comorbidity that could put the other patients at risk (infectious disease). C.) is not the correct answer because all patients do have the right to be free from excessive or unnecessary medication D.) is not the correct answer because all patients do have the right to the least restrictive environment E.) is the correct answer as well because patients are not able to leave at their own free will and can be involuntarily committed. Even if a patient comes in voluntarily, a patient must be deemed safe to not harm themselves or others before they are discharged.

Chris J. is a 17 year old high school student who has been dealing with depression. He is often bullied by his peers at school. He states that "nobody wants me here" and "I can never made my parents happy". Chris locks himself in his bedroom and plays video games for endless hours when ever he can. A.Impaired social interaction r/t poor social skills B.Social Isolation r/t factors contributing to the absence of satisfying person relationships C.Anxiety r/t mental status D. Risk for self harm r/t feeling hopeless

B is Correct. Nanda states there are many factors contributing to social isolation but the two key factors here are when the patients states " I can never make my parent happy" and the combination of bullying and stating "nobody want me here. Expresses feelings of rejection, inability to meet other expectations, being imposed by other, and obviously shows that he does not want social interactions any further are all in the Nanda for social isolation. Answer A in incorrect because impaired social interaction focuses more on the patients skills and ability to socialize, which is not stated in the situation. Answer C is not correct because the patient isn't experiencing anxiety. Answer D is incorrect becauise the patient never stated anything about self harm and it is not stated if the patient has a history of self- harm. (Wilkinson, 2014, pg. 713,714, 907).

Amy is an in-patient psychiatric patient with the following signs and symptoms: Flat affect, decreased energy, and is saying "just put me out of my misery". what is the best action for the nurse to take? A. Give her and antidepressant and check in with her later B. Ask if she has any suicidal thoughts and a plan to harm herself or others C. Refer Amy to a social therapy group D. Call the doctor to see if they are a "frequent flyer" before taking action

B is the correct answer due to the proper assessment of suicidal ideation and intent to commit suicide. According to Giddens, a nurse "should first assess emergency situations" when dealing with depressed patients (2012, pg. 323) A, C, and D are all interventions and should be considered after assessment.

A.B. is a 42 year old female who used to be a super upbeat and outgoing girl. Now A.B. states she, "doesn't have an appetite and wishes that something bad could happen that would take her off this planet." A.B. used to love getting ready and going out, but now she only has the will to shower 3-4 times a week. What would be the BEST nursing diagnosis for A.B. A. Ineffective coping r/t loss of mother B. Adult failure to thrive r/t depression C. Self-care deficit r/t having nobody to listen to D. Social isolation r/t inability to care for one self

B would be the best choice. After reading through these answers all of them have something that may be going on in A.B.'s life, but we are not given enough information in the question to assume some of these things have happened, like her mother passing. Using Maslow's hierarchy of needs thriving would be our highest priority before any of the other nursing diagnoses. Also in the NANDA book it talks about how when someone is failing to thrive they may; verbalize the desire for death, become anorexic or stating they aren't hungry, and their social skills may decrease and a related factor to this is depression. (Wilkinson, 2008, p. 226)

Sam is a nurse taking care of a patient, Julie, on his floor. This is a short-stay unit, so Julie has been here for about four weeks now. Sam noticed that when Julie first was admitted to his floor, she was restless, frequently needed to be reoriented to complete essential tasks, and had pressured speech. She was being closely monitored, given her frequent acts of reckless behavior. Upon being corrected in her behavior, Julie would yell about being a VIP in the hospital, and would threaten to have people fired. About a week into her stay, Sam noticed a sharp change in mood. Julie became withdrawn, sleeping through meals when allowed, refusing to leave her room. She started mentioning how dying would be easier than what she's dealing with, and how useless she was. Sam noted this to the doctor, who ordered a psych consult. Julie was diagnosed with a mental illness. What would Sam expect to see in Julie's chart as a diagnosis? A. Major Depressive Disorder B. Bipolar I C. Schizophrenia D. Anxiety, with psychotic features

B". Julie's symptoms point to her having a manic episode, followed by a depressive episode. This is a classic presentation of Bipolar I, especially the manic component. Though MDD could make sense, Julie had mania to begin with. This means that her symptoms are more appropriately defined as Bipolar. She does not fit the presentations needed for a diagnosis of schizophrenia, as her only "cross-over" symptom could be her delusions of grandeur. Julie also doesn't have the appropriate symptoms to be defined as having an anxiety disorder, or having true psychotic features. This leaves Bipolar as the appropriate answer, based on her full presentation.

You are the nurse providing patient education to James. The psychiatrist has prescribed James the medication escitalopram (Lexapro) which is an SSRI. What important pieces of information would the nurse want to provide to James before he is discharged from the inpatient psychiatric unit? (Select all that apply) A. Tell James that once his symptoms of depression decrease he may start scoring his tablets and taking half of his prescribed dose. B. Make sure James and his support system are aware that during the starting dose of a medication such as Lexapro, suicidal behavior may increase and it is important to monitor any significant mood changes and report them to his psychiatrist. C. Tell James is it important that he does not abruptly stop taking his medication as this may cause potentially hazardous side effects including irritability, dizziness, vomiting, nightmares, headaches, and increased suicidal behavior. D. Make sure James and his support system are aware that medications such as Lexapro may take 4 to 6 weeks to fully take effect and improve symptoms of depression. E. Tell James if he misses a dose of Lexapro one day, to not take the medication if he remembers later that day, and to take the missed dose the next day along with his regularly schedule dose.

B, C, D All answers in order, James should not alter his dosage of Lexapro unless approved by his psychiatrist or other healthcare professional. Educate him and his support system on increased suicidal risk during the starting dose of SSRI's, educate that James should not abruptly stop a medication such as Lexapro for the side effects listed. Educate that SSRI's may take 4-6 weeks to reach full effect. If you forget a dose of Lexapro, try to take it that same day if you remember, otherwise DO NOT take the missed dose the next day with your normally scheduled dose. AKA do not take more of the medication in one day than prescribed (Davis Drug Guide, 2015).

Sally is a 20-year-old female who is currently enrolled as a full-time nursing student. Last Thursday Sally took her medication administration check off for the second time and failed. Sally spent the weekend feeling hopeless, sad and defeated. Sally did not go to school Monday because of the way she was feeling. Sally's mother took her to the doctor because she was worried she was depressed and didn't want it to get worse. Her mother stated that she had experienced depression when she was a teenager but was better now. You are the nurse that is seeing Sally. What assessment would you perform on Sally and what would you say to her. (Select all that apply). A) You would do a Head-to-Toe assessment on Sally because you need to make sure nothing else is wrong with her. B) You would do a focused assessment, specifically a mental status assessment. You would also use one of the numerous standardized depression-screening tools. C)You would tell Sally "I understand not passing your checkoff must have been very hard on you. Can you tell me how you have been feeling and what thoughts have been going through your mind". D)Explain that although she is exhibiting some signs/symptoms of depression, depression is characterized by a persistently depressed mood lasting for a minimum of two weeks. Make a follow up appointment for two weeks to reassess her mental state. Tell Sally and her mother that if there is a significant change in her mood to come back in. E)Tell Sally that everyone fails, and she will get through it.

B, C, and D (Halter, Pg. 298, 303). According to Halter "Numerous standardized depression-screening tools that help assess the type and severity of depression are available". This a general assessment tool that can be used to screen for depression. Although she may be experiencing signs/symptoms of depression the mood has to be lasting for a minimum of two weeks to be diagnosed as depression.You wouldn't do a head to toe assessment, she is in there for depression so you'd focus the assessment. You also wouldn't just tell the patient that she is fine and just to get through it. You need to be understanding of what they are going through.

An elderly patient in long term care has just tried to commit suicide in her room. What are some nursing interventions the nurses could implement to help prevent this from happening again? (Select all that apply). A. Leave the patient alone and let her handle it herself. B. Monitor and manipulate environment to promote safety. C. Use hope inspiration. D. Manage the patient's mood.

B, C, and D. Answer B is important to make sure there is not anything for her to use to commit suicide. This will prevent her from committing suicide at all. Using hope inspiration enhances the belief in one's self and make her less likely to want to commit suicide. Mood management provides safety, stabilization, and comfort for the patient. This keeps there moods from being too elevated or too depressed, there by limiting the risk for suicide.

Depression is manifested in individuals due to the following (Select all that apply): a.) Overproduction of GABA b.) Having a constant negative, self-deprecating view of self c.) Dysregulation of norepinephrine and serotonin due to stressful life events d.) Genetics e.) All of the above

B, C, and D. According to Beck's Cognitive triad, if an individual looks at life in a negative way, possibly due to negative early life experiences, that individual is more likely to develop depression (b). The two main neurotransmitters involved in mood are serotonin and norepi, irregularities of each or both can result in depression (c). Multiple genes are involved in the development of depression, and it is a hereditary disorder (d). (Halter, Ch. 14, pgs. 252-255).

You are working as a nurse in the ICU. Your patient is a 76 year-old male who is intubated and being administered many medications including paralytics, pain medications, and sedatives. The patient has soft wrist restraints to prevent him from pulling out his endotracheal tube when he is not sleeping. As the nurse, you know that this patient is at high risk for developing delirium. What interventions can you apply to prevent the development of delirum in this patient? Select all that apply. A. Discuss with the provider about increasing the use of pain medications or sedatives because the patient seems to be agitated. B. Discuss with the provider about decreasing the use of pain medications or sedatives, because it can be related to a higher incidence of delirium. C. Discuss with the provider about discontinuing the use of restraints to promote patient mobility. D. Limit the patients time with visitors and family to promote rest. E. Cluster nursing activities to allow the patient to rest.

B, C, and E. The National Institute of Health explains that the use of sedatives, immobility, and sleep disruption are the highest risk factors for developing delerium in ICU settings. In order to prevent delirium nurses should prevent the use of sedatives unless absolutely needed, cluster nursing activities to promote rest and sleep, and to promote patient mobility with the decreased use of restraints, unless absolutely needed. The patient should spend as much time with family and visitors while still resting at night, because this helps keep the patient oriented (Brummel & Girard, 2013).

Your neighbor, Mrs. Smith, is recovering from the recent loss of her husband in the last six months. Which of the following situations would lead you to believe that she is in need of a mental health exam? (Select all that apply) A. Mrs. Smith sits on her porch every morning where her and her husband used to sit. She cries for a little while. Later, you see her at the store and she smiles at you. B. Normally, Mrs. Smith has her make up and hair done whenever she leaves the house, and prides herself on cleanliness. You see her at the grocery store and she looks like she hasn't showered or brushed her hair in weeks. Her clothes also appear to have stains on them. She seems to be having a hard time finding her way around the store. C. You invite Mrs. Smith if she wants a ride to church with you, since she normally goes every Sunday. She tells you that she is going to just do some religious practices at home today, but thanks you kindly for your offer. D. You take Mrs. Smith over some hot soup on a cold day. When she invites you in for lunch, you try to start some small talk. She gives you one word replies and acts completely uninterested.

B, D Rationale: In the book, there is a diagram that supplies some attributes of a mentally healthy individual. (Halter, pg. 2, Figure 1.1) Answer A: This demonstrates effective coping. Mrs. Smith is able to reminisce about her husband and have some sad emotions, but still makes an attempt to carry about her normal day-to-day life. Answer B: This does not demonstrate self-care. This answer shows that Mrs. Smith is neglecting herself, and shows a big behavioral change. This could also show that Mrs. Smith is exhibiting gross negligence. Answer C: This demonstrates spiritual satisfaction. Although it is odd that Mrs. Smith does not want to go to church, she still is showing that she is carrying out her spiritual and religious practices. Answer D: This does not demonstrate Mrs. Smith carrying out meaningful relationships or effective communication. Mrs. Smith is showing disconnect, and a disinterest in the situation.

Susie is a student nurse at her mental health clinical. Her instructor has assigned her to spend time with George, a patient with Bipolar II disorder. Susie has been asked to figure out what state George is in: manic, hypomanic, depressive, or in remission. What signs could Susie see if George was having a depressive episode? (Choose all that apply) A. George is rapidly changing topics during the conversation. B. George appears tearful during the conversation. C. George has pressured speech. D. George says leaving his room is "impossible". E. George notes that nothing seems fun anymore.

B, D, and E. "A" is representative of a manic/hypomanic episode - this would be flight of ideas. "C" is also indicative of manic/hypomanic episodes. "B" is an observable sign of depressed mood. "D" is a sign of loss of energy (anergia). "E" is a sign of diminished pleasure (anhedonia). This three are classic signs of a depressive episode. More symptoms would be necessary for it to be considered a full depressive episode, but Susie should be able to pick up on those cues.

A patient with schizophrenia is discharged from the mental health unit with the understanding that he will be put on a 1st generation antipsychotic medication. Which of the following can he expect his script to be written for? A: Latuda B: Haldol C: Clozaril D: Zoloft

B, Haldol. It is the only option that is a 1st generation antipsychotic. Rationale: Answers A and C are both 2nd generation antipsychotics. D, zoloft in an anti depressant

Anxiety is the most prevalent subgroup of mental illness in the United States. There are many things that put individuals at risk of developing anxiety, which of these factors increase the risk of an individual developing anxiety? (Select all that apply) A. Height B. Environmental C. Weight D. Genetics

B, and D (Giddens Pg 330) These are three factors that put an individual at risk of developing anxiety. Additional risk factors are temperament and physiological.

You are a nurse in the ER. Late one night, a mother comes in with her 5 year old son saying he fell from the banister and landed on his head on the hard wood floor. When looking at the patient, you can see something is seriously wrong. His eyes are open but he does not respond to you calling his name. He just stares ahead with a blank expression. When told to grip your hand he does so and is able to squeeze on command. After a CT scan of the patient's brain and neck along with an X-ray, the patient when calling his name, he responds with his name. What state of conscious is this patient? A) Lethargic B) Minimally conscious C) Vegetative State D) Coma

B, minimally conscious. Although it does seem the patient could possibly be in a vegetative state, looking at Giddens (2017) minimal conscious patients are able to respond to commands as well as respond to in a sterotypical fashion. The ability for the patient to grip the nurse's hand on command is a sign of the ability to respond to command.

The cerebrum (select all that apply) A. receives information from the spinal cord. B. Is the largest part of the brain and is divided into 4 lobes. C. Is responsible for pupillary reflexes and eye movement. D. Is responsible for mental activities and a state of conscious being. E. Deals with language and our ability to communicate.

B,D,E Rationale: A. False, the cerebellum receives information from the spinal cord. B. True, the cerebrum consists of frontal, parietal, occipital and temporal lobes. C. False, midbrain is responsible for those. D. True E. True

BB a 26-year-old male has been a reoccurring patient on your adult mental health unit. He was admitted after suffering a panic attack due to his anxiety, and the past 5 days he's been taking a Benzodiazepine for quick action to subset his panic and calm him down. He's been diagnosed with severe anxiety for two years now, and has been taking escitalopram (Lexapro) for the past year. Previously before the Lexapro he had been taking sertraline (Zoloft) the first year which had not worked for him so he was switched to Lexapro. Now the patient is reporting a low success with Lexapro. Since both SSRI medications have not been successful, they're looking to change the patient's medication again, what is the best choice for a long-term control over this patient's anxiety? A. fluoxetine (Prozac)-SSRI B. venlafaxine (Effexor)-SNRI C. alprazolam (Xanax)-Benzodiazepine D. phenelzine (Nardil)- MAOI

B. Explanation B. "Effexor is a serotonin norepinephrine reuptake inhibitor (SNRI) that is quite successful in the treatment of several anxiety disorders" (Halter). Compared to the rest of the choices this is the best choice of medication to try next for a patient that has not had success with SSRIs. This SNRI medication will act on serotonin and norepinephrine rather than just serotonin like an SSRI. Wrong answers A. Prozac is another SSRI, which would be a good choice for long-term control over anxiety but this specific patient has tried two different SSRI with no success this would be the wrong option. The patient's body may react better to SNRI and since two SSRI have been tested moving on to SNRI is the best choice. C. Xanax is a Benzodiazepine is a short-term control medication, and should not be used for long-term control. There are much better control methods with much less severe side effects than Benzodiazepines. D. Nardil the MAOI are considered a last resort medication. Although they may work the sided effects and possible life crisis with this medication put them on the back burner. They have many life-threatening interactions with other medication and foods especially tyramine. This would not be the best decision when an SNRI is also a choice.

Which of the following medication categories would be the first-line choice for treatment of Depression? A. Tricyclic antidepressants (TCAs) B. Selective Serotonin reuptake inhibitors (SSRIs) C. Norepinephrine-dopamine reuptake inhibitors (NDRIs) D. Monamine oxidase inhibitors (MAOIs)

B. SSRIs is the correct answer. In Gidden's, it is stated that, "The current first-line choice for the treatment of depression is the SSRI category because positive response rates are excellent and this category has the best rate of compliance as a result of its lower incidence of adverse effects". Other antidepressant drugs have a variety of unwanted side effects and restrictions.It has also shown have a greater positive response in patients. (pg. 323, Giddens, Concepts for Nursing Practice, Second Edition 2017)

Which is not a true statement about Lithium and it's relationship with Sodium? A. Patients should be advised not to drastically change their sodium intake. B. Sodium levels do not need to be monitored when taking Lithium. C. Lithium is contraindicated in patients who have a known sodium imbalance. D. Lithium ions alter sodium ion transport in nerve cells.

B. According to Pharmacology and the Nursing Process by Lilley, Collins, & Synder. Ch 16, page 255. A. Patients should be advised not to drastically change their sodium intake while taking lithium and to avoid overhydration as well as dehydration. B. Both sodium and lithium are monovalent positive ions, and one can affect the other. Therefore, the patient's serum sodium levels need to be kept in the normal range which helps to maintain therapeutic lithium levels. C. Contraindications to lithium therapy are relative and include dehydration, known sodium imbalance, and major renal or cardiovascular disease. D. Research indicates that lithium ions alter sodium ion transport in nerve cells, which results in a shift in catecholamine metabolism. Both sodium and lithium are monovalent positive ions, and one can affect the other.

JB is a 17 year old male that was involved in a MVA 2 weeks ago. When brought into the ED, his scans showed that there was swelling in his brain and is now 1 weeks post op after having partial craniotomy. When his mother entered the room and closed the door, JB shows a startled reflex. His eyes open, having slow spontaneous eye movements, but is unable to track or maintain eye contact. JB has sleep-awake cycles and sometimes he appears to be smiling. What type of altered consciousness is JB experiencing? a.Minimally conscious state b.Vegetative state c.Coma d. Persistent Vegetative state

B. According to (Maiese 2017) these are all descriptive signs of an individual that is in a vegetative state. Answer A is incorrect because a patient in a minimally conscious state are able to establish eye contact and respond to commands Answer C is incorrect because patients eyes remain closed and has no reflex responds Answer D, would not be the correct answer because with persistent vegetative state the symptoms last for 1 month or longer. JB symptoms have only lasted 1 week.

A client with schizophrenia has been started on medication therapy with clozapine (Clozaril). A nurse assesses the results of which laboratory study to monitor for an adverse effect related to this medication? A: Platelet count. B: White blood cell. C: Blood sugar. D: Liver function studies

B. Agranulocytosis may occur when a client is taking clozapine. Therefore, it can be monitored by evaluating the white blood cell count.

A nurse is assessing a patient for delirium. The nurse asks the patient several questions that include "does a stone float on water?" and "does one pound weigh more than two pounds?" What assessment tool is the nurse using? a) Mini Mental State Exam (MMSE) b) Confusion Assessment Method (CAM) c) Global Deterioration Rating Scale (GDRS) d) Diagnostic and Statistical Manual of Mental Disorders (DSM-5)

B. Giddens (2017) states that the MMSE consists of 11 cognitive tasks that cover the categories of time orientation, place orientation, immediate recall, short-term memory recall, serial 7's, reading, writing, drawing, and verbal/motor comprehension, the GDRS is a behavioral rating scale assessment, and the DSM-5 is a manual that categorizes impairments.

Your patient is in a coma from a traumatic car accident. They have a cervical spinal tract injury, and no brain injuries. What position do you expect your patient to be in? A) Decerebrate B) Decorticate C) Flaccid D) None of the above

B. Giddens (2018) states that the decorticate position is the position that can result from cervical spinal tract problems. Decerebrate or flaccid positions don't arise from spinal tract injuries, but are from brain injuries.

AK has been in a coma for a week now and has shown signs of decorticate posturing. Her family is concerned that she will never be her old self and is wondering what is going on. What are some of the postures the patient will be in? A. Stiff with straight arms B. Fingers are stiff and bent on chest C. Fanned out hands and fingers D. Legs bent and in fetal position

B. Giddens (2018.) has that in a decorticate posture the patient will have everything flexed and limbs are pulled in toward body. This is where the cervical spinal tract is damaged.

A 26 year-old male has come to the ED because he has not slept in 2 days and reports that he is "seeing things that he doesn't think are real". The patient is pacing the room, and speaking to you very quickly. As the nurse, you identify that the patient is most likely experiencing what form of altered consciousness? A. Bipolar disorder, hypomania B. Bipolar disorder, mania C. Delirium D. Generalized Anxiety Disorder

B. In bipolar disorder type I the patient alternates between phases of at least one episode of mania and major depression, and sometimes psychosis can be present in mania. In hypomanic episodes, psychosis is not present (Varcarolis & Halter, 2010).

Robert is a 34 year old male who has recently been experiencing Psychosis. He comes out of his room on the Monday morning you are scheduled to work and you notice that he has not showered, brushed his teeth, or combed his hair in several days. Although Robert does not seem to mind, it is repelling his peers from interacting with him. What is the best thing you could say to Robert to manage this situation? A: "Why do you not want to take a shower today?" B. "Perhaps if you brushed your teeth for one whole minute, I could go get your favorite game while you are gone and invite your peers to play?" C: "Is there something that is keeping you from wanting to brush your hair?" D: "I can arrange for you to have your favorite snack at breakfast if you can shower, brush your hair, brush your teeth, and put on a clean shirt before breakfast."

B. In order to achieve the best outcome in this situation, the nurse should "Suggest ways that improved hygiene will benefit the patient, perhaps greater acceptance by peers, increased privileges" (Halter, 2017, pg. 205). This request is manageable since you are offering the patient an incentive for better hygiene practices, while also making the request small and specific. Rationale: A and C would not be the best answers since they insinuate judgement and could potentially make the patient feel like they are not living up to your expectations. D would not be a practical request since it is asking the patient to do too many tasks at one time, rather than breaking it into feasible steps (Halter, 2017).

Patient was brought into the E.D. by police stating the patient tried to steal a shopping cart full of groceries, fled from them through the store and fell from climbing a shelf. Patient stated they were feeling great and that he was listening to the voice of God telling him to feed the homeless and become a god himself. Patient is most likely suffering from: A. Bipolar I Disorder B. Maniac Episode C. Bipolar II Disorder D. Cyclothymic Disorder

B. Maniac Episode Rationale: Halter, Chapter 13, pg. 223, states mania is a period of intense mood disturbances with persistent elevation, expansiveness, irritability and extreme goal directed activity or energy. As mania intensifies individuals may become psychotic and experience hallucinations, delusions and dramatically disturbed thoughts. Individuals may begin to hear voices, sometimes the voice of God. Even though this patient is likely suffering from Bipolar I Disorder, the BEST answer is Maniac Episode. The patient is currently experiencing this episode.

The nurse is doing a home health care visit to a 52 year old-male, J.D. J.D. has a history of diabetes. J.D.'s leg was amputated about 2 months ago, but the wound is healing poorly due to his love for eating fast food and his uncontrolled diabetes. Over the past two months, the nurse observes that J.D.'s is becoming more disheveled looking. He constantly apologizes for his appearance and for his messy house. The nurse knows that the best nursing diagnoses for J.D. is: A) Ineffective coping related to loss of leg B) Chronic low-self esteem related to shame and guilt of poor health C) Anxiety related to change in health D) Self-mutilation related to stress

B. On page 270 in the Varcarolis' Mental Health Nursing book, there is a table of nursing diagnosis for anxiety-related disorders. S/S of chronic low self-esteem include ashamed of the appearance of house due to hoarding, beliefs that others are disgusted with appearance, embarrassment of self. Ineffective coping would have S/S of the inability to attend social functions or avoidance behaviors. This could be a NANDA for J.D., however it is not the best one considering he enjoys going out and eating fast food. J.D. is not experiencing any anxiety attacks and that is why C is not the right answer. D is not right because J.D. is not showing any signs of self mutilation.

A nurse is giving instructions to a client taking risperidone (Risperdal). The nurse advises the client to do which of the following? A: Take it on an empty stomach. B: Change positions slowly. C: Get a daily source of sunlight. D: Discontinue medication once they start feeling better

B. Risperidone (Risperdal) can cause orthostatic hypotension so instruct the client to change positions slowly to avoid it.

Alexa Sumption Alexa Sumption Feb 15, 2018 Feb 15 at 9:35pm Manage Discussion Entry J.S. has been battling with bipolar disorder for the last 2 years. She was recently admitted to the hospital for not taking her medications and is currently in a manic state. While you are assessing the patient she calmly states "You sat on my cat, cats out of the box. New pair of socks. Do you have any rocks? I bought new stocks." You understand that the patient's statement represents which of the following speech patterns: A: Pressured speech B: Tangential speech C: Clang associations D: Loose associations

C According to Halter Chapter 13, clang associations are defined as "the stringing together of words because of their rhyming sounds, without regard to their meaning" (pg. 228). J.S's statement "You sat on my cat, cats out of the box. New pair of socks. Do you have any rocks? I bought new stocks" does not make any sense and the patient appears to be rhyming, or using clang association, to continue her thought(s). Choice A (pressured speech) is incorrect because J.S's speech is not fast or frantic and she is not displaying an inappropriate sense of urgency which is associated with pressured speech. Tangential speaking is defined as "trying to make [a point] and never find[ing] it again" (pg. 228), J.S. statement did not have a clear topic, meaning, or point so the answer tangential speech is innocent. Finally, in order for J.S's statement to be considered "loose associations" the statements would have to loosely be connected and/or related to each other and appropriate for the conversation (pg. 228), however, her statement was only connected through rhymes.

A.B. has a history of diabetes but has not been treated for it. She has been in the hospital for the last two days trying to get her A1C and glucose levels back to normal. The nurses start to notice she is having major fluctuations in her mental status and realizes she is delirious. Along with that the patient is also having visual hallucinations that her family is in the room with her. The nurses recognizes the patient has secondary psychosis related to the delirium. What is the best way to get rid of secondary psychosis? a. Medically treat the psychosis b. Reassure the patient that the hallucinations aren't real c. Treat the underlying illness that could be causing the delirium d. Try to get a better understanding of what the patient is seeing and talk to them about it

C In the Giddens book it talks about how in secondary psychosis the underlying medical illness needs to be treated first because this has a great chance of getting rid of the psychosis completely and that is why C is correct. A is wrong because medications could make the psychosis worse. D is an okay answer because that is good therapeutic communication with the patient; however, it won't get rid of the psychosis therefore it is not the best answer. As for answer B, that is not good therapeutic communication or helpful for getting rid of psychosis.

A patient has been admitted to an acute mental health unit and was diagnosised with bipolar type two and is in a hypomanic stage. While the nurse is doing the mental health assessment, what thought process would be expected? A) Coherence and auditory hallucinations B) Depersonalization and flat affect C) Loose association and flight of ideas D) Clear thinking

C Rational: The reason why A is incorrect is because coherence is not a thought process someone that is manic would experience. Also auditory hallucinations would not be seen in a hypomanic stage as well as hallucinations are perceptions not thought process. B is incorrect because neither depersonalization and flat affect are thought process. C is the correct answer because both loose association and flight if ideas are both thought process that are seen in bipolar disorder. D is incorrect because if someone is in a hypomanic stage they will not have clear thought process (Halter, p224-226,2018).

Which of the following does not have seeing dead people and believing you are a ghost buster as a possible symptom? A. Delirium B. Schizophrenia C. Eating disorders D. Substance abuse

C Rationale: Our Giddens book (Concepts for Nursing Practice) states that Delirium, Schizophrenia, Major depressive disorder, major neurocognitive disorder and substance/medication induced psychotic disorders can all produce symptoms of psychosis. Psychosis involves hallucinations, delusions and illusions; believing you are a ghost buster is a delusion and seeing dead people is a hallucination.

What factors do not affect the severity and progression of mental health and well-being? A.) Individual attributes and behavior B.) Environmental factors C.) Eye color D.) Social and Economic circumstances

C Rational: Eye color does not influence a patients mental health and well-being. Environmental factors, individual attributes and behaviors, and social and economic circumstances all encompass mental health and well-being.

Which of the following is the most accepted explanation for mental illness? A. Freud's Drive Theory B. Peplau's Theory of Interpersonal Relationships C. Diathesis-stress model D. The DSM-5 criteria

C Rationale: Halter states in chapter one "A diathesis-stress model—in which diathesis represents biological predisposition and stress represents environmental stress or trauma—is the most accepted explanation for mental illness." (Halter, 6). The DSM-5 provides the necessary criteria for professionals to diagnose patients, but does not explain mental illness as a whole. The other two choices are unrelated to mental illness directly.

Which individual has the least potential to recover from a psychotic episode without clinically significant sequelae? A. 88-year-old woman with dementia B. 47-year-old male alcoholic C. 34-year-old male with schizophrenia D. 24-year-old presenting signs of delirium

C Rationale: Those with a primary etiology of psychosis are more likely to manifest subsequent episodes of psychosis due to the predisposing genomic variation and subsequent neural circuit compromises. People with secondary psychosis have greater potential to recover from a psychotic episode without clinically significant sequelae.

A patient diagnosed with psychotic major depression states he hears voices commanding self-harm and that he must carry out this task. The patient refuses to commit to developing a plan for safety. What should be the nurse's priority intervention at this time? A. Obtaining an order to put the patient in seclusion B. Tell the patient that the voices are not real and that it's all in his head C. Placing the patient on one-to-one observation while monitoring suicidal ideations . D. Encouraging client to express feelings related to suicide

C -> Correct. When it comes to someone that is suicide, safety is important. Assessment of suicide risks or ideations should be an ongoing process to see if symptoms improve or get worse

You are the nurse on a psychiatric unit and are taking care of Mrs. Jones who is a 55 year old female who was recently admitted to rule out depression. Prior to admission Mrs. Jones' did research on the internet about depression and came across a website that mentioned the Diathesis- Stress Model and its relation to depression. Mrs. Jones is very interested in her care and wants you to explain this to her. Which of the following is the best response from the nurse? A) " I don't feel you would understand the concept and it won't benefit your care to learn about it" B) "I am really glad you want to know more about this model but only a physician can explain this to you" C) "This is a two part model that focuses on physiological vulnerabilities including gene tic predisposition, biological makeup, and personality structure. This part is referred to as the diathesis part. The stress part is talking about life events that impact you. If you have any other questions please let me know" D) "I will tell your therapist that you want to know about this and they can talk to you"

C Reasoning: As a nurse you are able to answer your patient's question regarding patient education. Referring them to a physician or therapist would be incorrect as answering this question is in a nurse's scope of practice. Lastly C is correct because "The Diathesis- Stress model of depression takes into account the interplay between genetic and biological predisposition toward depression and life events. The physiological vulnerabilities such as genetic predipsoisiton, biochemical makeup and personality structure are referred to as diathesis. The stress part of this model refers to the life events that impact individuals" (Halter, 2018, pg. 254). The information in C would correctly be explaining this to the patient.

Thomas Harms Thomas Harms Feb 12, 2018 Feb 12 at 10pm Manage Discussion Entry Kevin has Bipolar I disorder. He is currently in a manic state, and claims he is surrounded by demons who are trying to steal his soul. He is currently running through an outdoor mall, screaming at the bystanding pedestrian "demons", trying to scare them away by threatening them with a pipe. He is visibly frightened and very anxious. The most important nursing diagnosis for Kevin is: A. Impaired social interaction B. Altered thought process C. Risk for other-directed violence D. Risk for self-harm

C s/s of "risk for other directed violence" include "Alteration in cognitive functioning, impulsiveness, sexual advances, threatening violence, psychotic disorder (Halter 230)." Indivuals with bipolar I disorder "may become psychotic and experience hallucinations, delusions, and dramatically disturbed thoughts" (Halter 223). In this state, they can be impulsive and violent, without being aware of the harm they may do to themselves or others because of their delusions and psychosis. Though the other NANDAs do apply, given his psychosis, delusions, and threats against others, they are not the priority.

Mike was picked up by the police and brought to the hospital due to public intoxication. Pt. states they were "at the casino and having a great time" when all of a sudden he got kicked out. He also states that he "has been feeling low" lately until "God told him he would feel better if he went to the casino." He smells and has a disheveled appearance while being noticeably underweight. Which is the best nursing diagnosis for this pt? a. Self care deficit b. Risk for other directed violence c. Risk for injury d. Sleep deprivation

C (Halter, 2018) The best nursing diagnosis for this pt. is the rick for injury because it states in the the text that "suicide accounts for 5% of deaths among women and 10% of deaths among men." Since he is just coming out of a depression state and is self medicating with alcohol, this leaves him more prone to act on his feelings. He now has the energy to harm himself as he is in his manic stage and hearing "God" while spending all of his money at the casino.

Chelsea was separated from her mom in a large crowd as a 7 year old. She is now 17 years old and is afraid of large crowds as well as places that draw large crowds - from football games to concerts to parades and especially amusement parks or carnivals. Often, she does not go near these places, and when she does she feels crushing anxiety that she desperately tries to contain from her friends and family. She wants to be comfortable enough to be able to go to her Senior Prom in May and is seeking help from your clinic. Which of the following is the priority nursing diagnosis for Chelsea? A) Post-traumatic stress disorder related to being separated from her mother in a large crowd B) Anxiety related to situational and maturational crises as evidenced by avoidance of large crowds C) Fear related to a learned response from her childhood as evidenced by avoidance of large crowds D) Situational low self-esteem related to failure and lack of recognition as evidenced by inability to attend events

C Rationale: C best illustrates Chelsea's fear of large crowds and relates it to the causative event in her life of being separated from her mom in a crowd as a child. This behavior is a learned response from that time frame that has been reconfirmed throughout her life to this point. A utilizes a medical diagnosis, which makes it incorrect. B while Chelsea does experience anxiety, it is not due to a maturational crisis, as this is not a new behavior but rather a behavior that has been going on in specific situations since her childhood. D does not fit Chelsea's situation as she recognizes that she needs help and wants to improve. While her esteem may suffer in certain situations, this is not what brought her in to the facility to seek treatment.

Nicole Nothelfer Nicole Nothelfer Feb 15, 2018 Feb 15 at 6:54pm Manage Discussion Entry A new 19-year-old admit in the emergency department where you work has been screaming, crying, hyperventilating and repeatedly stating she is scared to die for the past half hour. She is physically fine and upon admittance her friend reports that nothing physically had happened to her. You've administered her first dose of Xanax 10 minutes ago. Collected her vitals BP 160/90, pulse 112, respirations 32, oxygen 100%, temperature 98.6. With further look at her medical records you learn two months ago she was diagnosed with generalized anxiety disorder and has a current prescription for fluoxetine (Prozac). As her nurse with this information what interventions are appropriate at this time? (select all that apply) A. Medication adherence: Assess whether the teenager is accurately taking her medication as prescribed. B. Spiritual Support: Assisting the patient to feel balance and connection with greater power. C. Emotional Support: Provision of reassurance, acceptance, and encouragement during times of stress. D. Presence: Being with another, both physically and psychologically, during this time of need.

C & D. Explanation The situation that is presented is anxiety and the patient is in a state of panic. The nurse first needs to help calm her down and help relieve the panic she's experiencing. The best choices to bring her out of the panic attack are to provide emotional support and just simply being a presence for the patient. By staying with her and giving her attention she will realize you care and this may provide her comfort. In a time of panic comfort is the highest priority second to safety. Using short simple sentences, reassurance and brief directions the nurse can complete an intervention for this immediate need of relief. Rationalization is not a realistic goal for the patient. As stated by Halter, "Someone in a state of panic is unable to process what is going on in the environment and may lose touch with reality." The patient's reality is obviously disturbed. Spiritual support nor medication adherence should not be addressed as she is not in the correct mental state to communicate that information. The patient's adherence to her Prozac prescription is not going to help her current panic attack so that is a situation that can be addressed later once her mental state is controlled. The medication that will help her situation has already been given(Xanax), and is not dependent on the patient's medication adherence but is currently the nurse's responsibility. The patient is a new admit with an on going panic attack religion had not been discussed beforehand so the nurse should not project spiritual anticipation.

Jaime is a 33 year old single mother to four young children. She was just involuntarily admitted to your unit for psychosis. At the time of admission, Jaime was hysterical over the thought of being diagnosed with schizophrenia because she has been "hearing voices" for the past month. During your initial assessment, Jaime discloses that she is addicted to cocaine and has been abusing the drug for a little over a year. Her prior mental health history includes a single suicide attempt after the passing of her father six years ago, while her prior medical history includes left knee arthroplasty two years ago. Jaime repeatedly tells you that she feels she has lived a fairly healthy life and does not understand what has made her psychotic. Which of the following most likely caused the sudden onset of Jaime's psychosis? A. The passing of Jaime's father B. An imbalance of glutamate in Jaime's brain C. Jaime's cocaine use D. Jaime having to raise four young children by herself

C according to the dopamine theory found on page 194 in the Halter (2017) textbook. "Amphetamines and cocaine can induce psychosis in people without schizophrenia and can also bring on the disorder" (Halter, 2017, p. 194). When dopamine-2 (D2) receptors are blocked there is symptom reduction of both hallucinations and delusions, thus dopamine appears to play a significant role in psychosis (Halter, 2017, p. 194). Cocaine is a drug that increases the amount of dopamine in the brain. A&D are incorrect because while they may be difficult situations that occurred in Jaime's life, they are not the most likely cause of psychosis. B is not correct because several neurotransmitters play a role in the diagnosis of psychosis (Halter, 2017). Also, "substance use and medical conditions should always be ruled out before a primary diagnosis of schizophrenia or other psychotic disorder is made" (Halter, 2017, p. 192).

L.H. is a 25 year old male who has voluntarily entered himself into the behavioral health unit and you are the nurse in charge of his care today. He is diagnosed with depression and you are trying to talk with him to see where his mood is, but he seems very withdrawn and not open to engage in conversation. What are the most effective ways to speak with a patient who is withdrawn and depressed? (CHOOSE ALL THAT APPLY) A. Asking direct questions to engage them in conversation B. Using sayings like "Things will get better or everyone gets down sometimes" C. Allow time for the patient to respond D. Ask about suicide plans

C and D are correct according to Halter because "(C) Slowed thinking necessitates time to formulate a response," and " (D) People often experience relief and decrease in feelings of isolation when they share thoughts of suicide" (Halter, 2014, p. 262).

Which of the following individuals is at the highest risk for suicide? A. DP, age 15, no physical or mental health problems but experiencing loneliness B. PL, age 72, recently diagnosed with schizophrenia and stays in his room all day except meals C. AB, age 22, diagnosed with major depression experiencing more energy the past few days D. HW, age 55, diagnosed with bipolar type 2 with one past suicide attempt

C because this patient is experiencing more energy lately. According to Halter (2014), two thirds of those who commit suicide are experiencing depression at the time. Patient C is experiencing major depression as well as having more energy the past few days. Patients who are having suicidal ideations are more likely to act on their thoughts when they are experiencing more energy because they have the energy to put into their plan. Patient A is not our highest priority patient because a patient with mental health disorders is more likely to commit suicide than someone experiencing loneliness. Patients with schizophrenia are 50 times more likely to commit suicide but with this patient staying in his room all day except for meals, he seems to not have as much energy that he would need to put forth. 15% of patients with bipolar will commit suicide and while this patient has attempted once before making him at a higher risk to attempt again, the patient who is having more energy is most likely to act and is our highest priority. (pg. 481)

A patient was assessed and stated that they felt "everyone leaves me so what's the point of talking to anyone anymore". They haven't been getting along with their roommate and sometimes refuse to speak to the nurses. Which would be the best nursing diagnosis? A) Anxiety r/t poor personal relationships B) Defensive coping r/t denial of problems C) Social Isolation r/t an inability to engage in personal relationships D) Noncompliance r/t failure to make progress

C is the best answer because one of the key defining factors of social isolation according to the NANDA is "feelings of aloneness imposed by others" (Wilkinson, 2017, pg. 815). The patient has given up on talking to others because they perceive others as always leaving them and not wanting to engage with them.

A client with a bipolar disorder exhibits manic behavior. The nursing diagnosis is disturbed thought processes related to difficulty concentrating, secondary to flight of ideas. Which of the following outcome criteria would indicate improvement in the client? (choose all that apply) A The client verbalizes feelings directly during treatment. B The client verbalizes positive "self" statement. C The client speaks in coherent sentences. D The client can hold a conversation about one particular topic for an extended period of time.

C, D Rationale: A client exhibiting flight of ideas typically has a continuous speech flow and jumps from one topic to another. Speaking in coherent sentences is an indicator that the client's concentration has improved, and his thoughts are no longer racing. The remaining options do not relate directly to the stated nursing diagnosis.

A nurse notes that a client with schizophrenia and receiving an antipsychotic medication is having uncontrolled movement of the lips and tongue. The nurse determines that the client is experiencing? A: Hypertensive crisis B: Parkinsonism C: Tardive dyskinesia D: Malignant syndrome

C, Tardive dyskinesia is characterized by uncontrollable involuntary movements of the body and extremities (especially of the face, lips, mouth, tongue, arms or legs).

Which of the following would not be an appropriate outcome for a patient with a panic disorder? a. Patient will be able to recognize symptoms resulting from anxiety b. Patient will use abdominal breathing immediately when anxiety is detected c. Patient will not experience any more panic attacks d. Patient will use positive self-talk, such as "I can control my anxiety"

C, because this is not an attainable outcome for the patient. The goal is to control the panic attacks and it is likely that they will still have panic disorder. A is an appropriate outcome because factual information and alternative interpretations can help the patient recognize distortions in thought. B is an appropriate outcome because breathing exercises break the cycle of escalating symptoms of anxiety. D is an appropriate outcome because negative self-talk contributes to anxiety. Cognitive restructuring is an effective way to replace negative self-talk (Halter, pg. 284).

What is true about people with OCD? A. The obsessions and/or compulsions the people have are voluntary thoughts or actions B. They are relieved immediately once they have their thought/compulsion C. People affected with OCD experience significant problems in their life related to their thoughts and behaviors D. There is no treatment for OCD

C, people with OCD know that their actions/thoughts are irrational but they cannot stop doing them. They often have hours a day devoted to doing these acts and this can have a bad effect on their family and friend relationships. While some individuals may get brief relief, the longer a person goes without treatment the thoughts and behaviors often get worse with each action. This just continues to effect their relationships and takes up more and more of their time, thus affecting their relationships more and more. There are treatments for OCD, but what may work for one person may not work for another. The first line treatments of ERP and medications will work for 70% of people, so it is just a matter of finding what works best for them.

Which of the following statements if made by a patient are the least likely to be indicative of a mental health problem on the mental health-mental illness continuum? (Select all that apply.) A.) "I feel as if I am at the bottom of a hole with no way of digging myself out no matter what I do." B.) "I feel so alive right now! I am going to buy a plane ticket right now to climb Mount Everest in nothing but my underwear!" C.) "My life hasn't been this stress free since I began working 40 years ago. Retirement feels great!" D.) "I am afraid to go out in public, I feel as if everyone is staring at me when I go to Meijer." E.) "Studying for exams can be stressful at times, especially when I have to work all week."

C,E Rationale: A: This statement infers that the patient is experiencing hopelessness possibly related to some form of depression, indicating a mental health problem on the continuum. B:This statement is likely to be made by a patient experiencing mania, which is also indicative of a mental health problem. C: This statement reveals that working was a stressor at one point in time. However, stress as a result of everyday life is not considered a mental health concern as long as it does not impair functioning. Furthermore, this patient is retired and clearly indicated that he has no stress currently. D: This statement reveals a severe anxiety problem and indicates an impairment of daily functioning, thus lacking a state of well being on the continuum. E: This patient indicates that studying and having a job is stressful. However, this stress is to be expected and does not indicate a mental health problem as long as it does not impair functioning long term.

All of these can be nursing interventions for a patient in depressive psychosis who are experiencing disturbed thought processes except: A. Use simple, concrete words B. Allow more time than usual for the patient to finish usual ADL's C. Help the patient to expedite important major life decisions D. Help the patient and their family members to structure an environment and daily routine

C. Answer C is correct because you do not want your patient to be making any major life decisions while they are in such a depressed state. To make a life decision requires optimal psychophysiological functioning, but when depressed, cognitive functioning decreases. Answer A is something you would use as a NIC because using large, complex works slows thinking and difficulty concentrating impair comprehension, thus you want to use simple, concrete words. Answer B is something you would do as a nurse because usual tasks might take long periods of time; if you demand that the patient hurry with their ADLs then you will increase their anxiety and slow down the ability to think clearly. Answer D is useful for depressive patients because a non-demanding repetitive routine is easier for the patient to remember and follow. (Halter- Ch. 14, pg. 202)

AM was just released from the ICU for a brain injury, due to a baseball getting thrown at her head. Pt was placed in the recovery unit and appeared to be aware, but with fluctation. What level of consciousness does the patient appear to be in? a) persistent vegetative state b) alert c) minimally conscious state d) coma

C. Rationale:A person that is in the minimally conscious state shows signs of awareness, but it fluctuates.

Which of the following is the term that describes a patient that is experiencing problems with the cervical or spinal tract, the cerebral hemisphere, and the arm are shaped like "c's"? A. coma B. Stupor C. Decorticate D. Vegetative state E. Decerebrate

Correct answer is C. Answers A B and D describe different levels of consciousness while C and E describe posturing that can occur with damage to the brain. Decorticate describes a patient with arms in the flexed shape that tend to form into "C" shapes (Giddens, 2018).

There are four different levels of Anxiety in Peplau's model. The levels of anxiety range from mild anxiety to moderate anxiety, severe anxiety, and panic. Each level of anxiety has different signs and symptoms present. If a patient came to the doctors office with complains of Concentration progressively narrowed, Severe impairment of attention, Severe cognitive impairment, Physical symptoms, and Emotional symptoms what level of anxiety would this individual be experiencing? A. Mild anxiety B. Moderate anxiety C. Severe anxiety D. Panic

C. (Sever anxiety). Concentration progressively narrowed, Severe impairment of attention, Severe cognitive impairment, Physical symptoms, and Emotional symptoms are all classified under the category of Severe anxiety on the chart labeled Peplau's Four Levels of Anxiety (giddens Pg 327).

A 30 year old, female patient with a diagnosis of major depressive disorder states, "Ever since my husband died, I feel depressed. He was my best friend, the father to my children, and my biggest supporter. I don't know how to be happy without him." What is the nurse's best response? a. "I'm sorry to hear about the loss of your husband." b. "That is awful. My brother died last year and it was difficult for me too." c. "That must be hard. You mentioned that you are depressed. When you feel depressed, what thoughts go through your mind?" d. "I'm sorry for your loss. Have you tried finding happiness in other activities that you enjoy to distract you from what you are feeling?"

C. According to Halter, about 15% of individuals with depression commit suicide so it is important to assess for suicide potential. In order to approach initial suicide assessment, Halter suggests using the following statements/questions: "You said you are depressed. Tell me what that is like for you." or "When you feel depressed, what thoughts go through your mind? " These questions are more open-ended and allow the patient to give more detailed answers which allows the nurse to collect more assessment data. The reason the other choices would not be the best answer is that they do not allow or provoke the patient to give a detailed response which prevents the nurse from gathering the necessary assessment data (Halter, 2014, pg. 255).

As a student nurse you are talking with your patient who has Major Depressive Disorder at your mental health facility. You want to find out if the patient is having suicidal ideations or not. Which of the following is the best way to ask about thoughts of suicide? A. "Do you have a plan for suicide such as overdosing on medication?" B. "I know you say you've been feeling down, but you don't have thoughts of killing yourself do you?" C. "Have you ever felt that life is not worth living?" D. Don't ask unless they bring it up first.

C. Halter states this line ("Have you ever felt that life is not worth living?") as a good, specific question to ask when wondering about suicidal ideation of a patient. It addresses directly what you are concerned about. A. Is wrong because it's offering an option or idea to them. They may have a plan for suicide but you don't want to fill their mouth with words, nor offer up an idea or way to commit suicide. B. Is wrong because the question comes off as judgemental and they may then not want to discuss with you their thoughts because they are afraid you are looking down upon them. D. Is wrong for a couple reasons. First, you want to know if these thoughts of suicide are going through their head in order to treat them correctly and have percautions to keep them safe. It is a responsibility, just as asking a cardiac patient about chest pain is a responsibility. Secondly, as it says in Halter, it is most often a relief for someone who is thinking about suicide to finally talk to someone about their feelings of loneliness and despair and can lower their feelings of isolation. The above question information was referenced from "Foundations of Mental Health Nursing, Halter, 7th edition, pages 485-486

XY presents to the ED complaining of anxiety. She states she can't breath, she has has extreme chest tightness, and feels "as though she is having a heart attack". XY reports that she is currently a single mother of two, is going through the process of a divorce, and is also taking care of her elderly parents. While XY is describing her situation, her blood pressure and heart rate rapidly rises from 130/80 to 150/90. What is the most appropriate/best nursing diagnosis for XY? a.) Anxiety (severe) related to patient chest tightness and inability to breath as evidenced by patient going through divorce, being a single parent, and taking care of elderly parents. b.) Ineffective coping related to divorce and caregiver burden as evidenced by patient's record of personal life and elevated BP and HR. c.) Anxiety (panic) related to loss of significant others, single parenting, and caregiver burden as evidenced by patient description of anxiety, current situational issues in patient personal life, and elevation of BP and HR. d.) Anxiety related to situational crisis as evidenced by patient personal life description and increased vital signs.

C. The nursing diagnosis is complete, in the correct order as well as fully descriptive to the situation as a whole, also the patient is experiencing anxiety at the panic level. A is not in the correct order, the related factors are the evidenced factors, and vice versa. B is not the best diagnosis; anxiety would be the best diagnosis in this situation. D does not describe the level of anxiety the patient is experiencing, and it is not as detailed as the others in explanation. Halter, pg. 293, describes different s/s for anxiety nursing diagnoses ranging from mild to panic anxiety. Panic anxiety includes marked changes in baseline behavior, functional impairment, and emotional and behavioral dysregulation. (Halter, 2018)

Michael is a 17-year-old male that has recently been diagnosed with major depressive disorder and had been admitted to a mental health facility after expressing feelings of suicide to his therapist. He is currently showing signs of depression and says, "He hates his life, and sees no point in living anymore". As a nurse, what is the best action to do next? A. Take Vital Signs and ask what medications he is taking B. Ask him why he hates his life C. Convince him that his life isn't that bad D. Ask him if he has any suicide thoughts

D Rationale: Giddens (2018) states that it is a priority to address patients when are at risk for suicide and they should not be left alone.

A.B. has been diagnosed with psychosis. He has been on medications for a while, but they have not been working as well as he had hoped. Before being prescribed additional medications, he asked about other ways to help with his psychosis. There different therapies he could try. Which of the following would not work well for him? A. Electroconvulsive therapy - electrical currents pass through the brain with intent of inducing seizure activity, which may cause changes in brain chemistry B. Cognitive Enhancement therapy - focuses on improving mental stamina, active information processing, and learning how to negotiate unrehearsed social challenges C. Cognitive-Behavioral therapy - can improve medication adherence, insight, and social functioning D. Cold Laser therapy - uses levels of light to stimulate healing

D Rationale: Giddens recognizes electroconvulsive, cognitive enhancement, and cognitive-behavioral therapies as nonpharmacologic therapies for psychosis. In addition, Giddens also identifies social skills training and family focused therapy as possible therapies. However, Cold Laser therapy is not used for psychosis and is used for tissue repair and relief of pain and inflammation.

Mr. Peterson, 35, is admitted for bipolar illness, manic phase, after assaulting his landlord in an argument over Mr. Peterson is staying up all night playing loud music. Mr. Peterson is hyperactive, intrusive, and has rapid, pressured speech. He has not slept in three days and appears thin and disheveled. Which of the following is the most essential nursing action at this time? A Providing a meal and beverage for Mr. Peterson to eat in the dining room. B Providing linens and toiletries for Mr. Peterson to attend to his hygiene. C Consulting with the psychiatrist to order a hypnotic to promote sleep. D Providing for client safety by limiting his privileges.

D Rationale: Food and fluids are necessary. However, Mr. Peterson's hyperactivity does not allow him to sit quietly to eat. Finger foods "on the run" will provide needed nourishment. When hyperactivity decreases, then approach Mr. Peterson's. regarding hygiene and grooming needs. Medications will be ordered. However, a thorough evaluation must be done first. Mr. Peterson has been assaultive with the landlord and it is reasonable to expect that he may be with peers and staff. His mental illness produces a hyperactive state and poor judgment and impulse control. External controls such as limiting of unit privileges will assist in feelings of security and safety.

A 59 year old male patient comes into the ED. In the ED, he states "I feel alone" "I feel like no one cares about me, I just want to die". What as a nurse do you know would be the highest priority in this situation. A. Take VS and ask what medications they are taking B. Ask past suicide attempts C. Ask the patient if there are any stressors in his life D. Establish trust with the patient and ask the patient if he had a plan to harm himself

D Rationale: The highest priority in this situation is to establish the trust with the patient so that the nurse can figure out if the patient had a plan to harm himself. The rest of them could be questions or things that could be done later on, but is not the highest priority. Giddens (2018) states that it should be considered an emergency when patients are at risk for suicide or violence and should not be left alone. Behavioral signs may warn of suicidal ideation which includes writing or creating art about death, giving away prized possessions and joking about death, dying, suicide, or leaving (pg. 323, 324).

Maria is an 18-year-old girl that has recently lost her father. She was driving him to work when they were in a car accident in which her father died and she lived. She has no siblings, and her mother was never really in the picture after their divorce. Maria has lost 21 pounds in one week. She is severely depressed and feels as if she has no reason to live. She feels as if her father's death was her own fault, and with that she has plans to harm herself in order for her to get the justice she deserves for her father's death. Which nursing diagnosis would be the top priority for Maria?\ A. Loneliness related to emotional and physical isolation as evidence by the patient not having family to turn to in time of need. B. Hopelessness related to prolonged stress and deteriorating physical condition as evidence by the patient losing 21 pounds in one week and feeling as if she has no reason to live. C. Complicated grieving related to death of a significant family member as evidence by patient feeling as if her father's death was her fault. D. Self-directed violence related to the death of her father and a feeling of guilt as evidence by the patient losing 21 pounds in one week along with her stating that she has plans to harm herself. E. Chronic sorrow related to unresolved grief as evidence by the patient becoming severely depressed after the death of her father.

D - would be the correct answer because of the fact that this is the most prevalent to the patient at this time. According to Maslow's hierarchy of the needs, the first concern for this patient is to assess the fact that she is at risk for harming herself. The NANDA book talks about how this diagnosis is very high in importance, although the others can apply to this patient as well (Wilkinson, 2017, pg 59).

April has been seeing a psychiatrist (Dr. Crane) for two months and during her interviews he's noted that her speech is continuous and she jumps from topic to topic. Dr. Crane has been told that she has also been observed to be nuisance to others by taunting them with loud or crass dialogue, hopping from one person to another incessantly and sometimes displays provocative behaviors. What would be the best nursing diagnosis for this individual? a. ineffective coping r/t cognitive limitation b. risk for injury r/t constant flurries of activity c. disturbed thought process r/t borderline personality disorder d. impaired social interaction r/t recurrent episodes of mania

D According to Halter (2010), the nursing diagnosis most applicable to "constant motor activity, annoyance or taunting of others, loud or crass speech, and provocative behaviors (p. 289)" is impaired social interaction.

M.R. has a history of being sexually abused as a child by the youth group leader at his church. He is now 34, works from home and has isolated himself from his family. He finds out that the same group leader is still abusing boys in the youth group. After years of never telling anyone about the abuse, M.R. has flashbacks that caused overwhelming emotions to come back. He is extremely angry and intends to confront the group leader about the abuse. What is the most appropriate nursing diagnosis for M.R.? A. Post-trauma syndrome related to criminal victimization AEB sexual abuse as a child B. Anxiety related to stress AEB anger, confrontational intentions, and intrusive memories C. Ineffective coping related to stress AEB withdrawal from support system. D. Risk for Violence: Other-directed related to history of child abuse

D is correct because the risk of others being harmed is priority. A is a great diagnosis for this patient because he definitely is experiencing PTSD, but his life is not at risk right now. B is not correct because the priority in this situation is the safety of everybody, so B is only addressing his emotional state. C isn't correct because the ineffective coping has led to his risk for violence which prioritizes the safety aspect of the situation.

Ellie is a 17-year-old female that was sexually abused by her step father when she was 12 years old. Ellie now suffers from PTSD as a result of the sexual abuse. Ellie in unable to have meaningful relationships which makes her feel extremely lonely since all of her friends have boyfriends that they hang out with a lot instead of hanging out with Ellie. Ellie also feels very uneasy around males that she does not know. Ellie also feels that she is not pretty or skinny enough. Which of the following would NOT be considered a possible NANDA for Ellie? A. Anxiety B.Social isolation C. Chronic low self-esteem D. Grieving

D would be the correct answer because Ellie is diagnosed with PTSD as a result of the sexual abuse in her past. Ellie is anxious around men that she does not know, has chronic low self-esteem since she feels that she is not pretty or skinny, and she is socially isolated since all of her friends have boyfriends. This information makes A, B, and C possible NANDAs for Ellie. All of these are also possible NANDAs for someone diagnosed with PTSD.

Which patient is most likely to experience an altered conscious state? A. A 98 year old male whose wife passed away 5 years ago B. A 19 year old male who has used cannabis in the past C. A 38 year old female who just gave birth and is now depressed D. A 76 year old female who presented to the ED severely dehydrated

D, because according to Giddens on page 341, health conditions such as electrolyte imbalance can cause altered cognition in anyone, not just older individuals (Giddens, 2017). Although Cannabis can cause an altered conscious state, this 19 year old male used it in the past. More than likely his cognition is not altered right now. `

An 80 year old male patient that has been diagnosed with hypertension for the past 10 years is at his yearly checkup with her PCP. Recently he has been having difficulty remembering familiar things, such as family members names, and is becoming worried about it. What information should the PCP provide to this patient regarding her concerns? A. "It is common for patients with high blood pressure to experience memory loss, so what you are experiencing is normal" B. "Hypertension can put a lot of strain on your heart, making it difficult for the rest of your body to function properly, such as your brain. C. "You are just getting old and this is normal" D. "High blood pressure puts excess strain on the brain cells, which can impact the way your brain works. I would like to examine you for signs of Dementia"

D. High blood pressure puts excess strain on the brain cells, which can cause neurons to degenerate and the synapses between them to be broken. Decreased neurons and synapses decreases the brains ability to communicate and function properly, which leads to signs of dementia like memory loss (Giddens, 2017).

A patient has just recieved his haloperidol medication for his psychosis. The nurse came back to reassess the patient 30 minutes later. When she begin to assess the patient she notices the patient exhibiting acute dystonia, tardive dyskinesia, and akathisia. What is the best explanation of these symptoms? a) Parkinson's disease b) hypomania c) depression d) extrapyramidal symptoms e) all of the above

D. Rationale: EPS can be a side effect of haloperidol.These side effects include; acute dyskinesias, acute dystonia, tardive dyskinesia, parkinsonism, akathisia, and neuroleptic malignant syndrome.

Your patient is a 52 year old divorced woman with no children. She has COPD (emphysema), and self-medicates with alcohol for her insomnia. You know that these conditions put her at a higher risk for which of the following: A. Parkinson's Disease B. Alzheimer's Disease C. Traumatic Brain Injury D. Cognitive Impairment

D. According to Giddens (2017), significant risk factors for developing cognitive impairment in women are lack of social support, insomnia, poor health status, and dependency. The correct answer is not A or B because those are both types of neurocognitive disorders. The correct answer is not C, although her risk for TBI may be higher because of how her conditions may affect her actions, she is at most risk for cognitive impairment based on the risk factors provided.

Louis has anxiety when interacting with others and has social anxiety. He went out with a group of friends to dinner and began to experience increased anxiety and his perception of the outside surroundings begin to decrease. As a coping mechanisms would be a beneficial coping skill. A. Suppresses the feelings of anxiety B. Drinks to intoxication C. Displaces negative emotions onto best friend D. Walks outside to reduce stimuli

D. The answers A,B and C are coping mechanisms but are not beneficial for the individual. Page 283 chart of coping mechanisms people often use to cope with anxiety.

A 33-year-old female has just recently lost their spouse of three weeks by MVA. During an interview the nurse notices that she has little to no eye contact, she has a flat affect, and seemed preoccupied. The patient stated "I haven't left the house much and I haven't been eating much either". When asked if a therapist would be helpful the patient responded "I don't really want to talk to anyone about this". Choose the best nursing diagnosis. a. Risk for self-mutilation r/t poor self esteem b. Imbalanced nutrition: less than body requirements r/t loss of appetite c. Social isolation r/t recent loss of loved one d. Anxiety r/t stress

D. According to Halter (2014), the best nursing diagnosis for this set of signs and symptoms is social isolation. A patient with social isolation can have a dull or sad affect, no eye contact, may be preoccupied with their own thoughts, and they may seek to be alone.

Jenny is diagnosed with Bipolar Type 2. She is recently experiencing a state of mania. Which of the following behaviors constitutes mania. A. Auditory Hallucinations with random periods of Command Hullicinations B. Flat affect with little interest in surrounding stimuli C. Sleeping for several hours and takes several hours to wake in the morning D. Increased energy and partakes in dangerous activities

D. Additionally, the presence of three of the following behaviors constitutes mania: Extreme drive and energy, Inflated sense of self-importance, Drastically reduced sleep requirements, Excessive talking combined with pressured speech, Personal feeling of racing thoughts, Distraction by environmental events, Unusually obsessed with and overfocused on goals, Purposeless arousal and movement, Dangerous activities such as indiscriminate spending, reckless sexual encounters, or risky investment.

All of the following are nursing interventions for a schizophrenic patient experiencing disturbed sensory perception except: A. Be alert for signs of increasing fear, anxiety, or agitation. B. Accept the fact that the voices are real to the patient, but explain that you do not hear the voices. C. Explore how the hallucinations are experienced by the client. D. Teach the client to ignore the voices, and to focus on their daily goals.

D. Rationale: Patient might be having hallucinatory activity, which can be frightening and patient might act upon command hallucinations. Validating that your patient's reality does not include voices can help the patient cast "doubt" on the validity of his or her voices. This can give the patient a sense of power that he or she might be able to manage the hallucinatory voices. You should never doubt a patient's thoughts, this can close off communication between nurse and patient.

MM is a male patient who underwent open heart surgery 2 days prior was transferred from ICU to the PCCU for recovery. The patient seemed to be recovering well but within the past 24 hours began to experience a sudden change in mental status. As the nurse for MM you determine a Confusion Assessment Method (CAM) exam needs to be done, what would you do for this CAM? A) Identifying a change in mental status other then the patients baseline B) Preforming a letter attention test (eg. SAVEHAART) C) Yes and No questions (eg. Does a stone float on water?) D) Command the patient to preform tasks (eg. Hold up this many fingers. Now do the same on the other hand) E) All of the above

E, All of the Above. Looking at Giddens (2017) all of these options are a part of the Confusion Assessment Method (CAM) that is used to determine delirium in a hospital setting. Along with all these parts there is another part which is to preform a Ricker Sedation-Agitation Scale (SAS) or Richmond Agitation-Sedation Scale (RASS) to determine the level of consciousness of the patient. If anything other then calm for either one, it is an alter level of conscious.

Patient QR has a history of stroke with left sided weakness.She had cataracts forming on both eyes. The patient is 82 years old and is admitted to the hospital for dehydration and is showing signs of mild delirium. Which of the following is the priority nursing intervention for this patient? A. Eye damage prevention B. CVA prevention C. Left Side rehabilitation D. Rehydration E. Fall prevention

E. According to Wilkinson, 2016, the most important intervention for this patient would be to decrease any fall risk while in the hospital. Answer D is also extremely important, but can only be achieved over a course of time while the fall risk prevention can be implemented immediately and is of utmost importance with the patient's history of weakness and poor vision.

Lithium is a common mood stabilizer used in the treatment of bipolar disorder. What neurotransmitter is regulated with the use of lithium therapy? a. serotonin b. glutamate c. norepinephrine d. acetylcholine e. dopamine

Giddens states that "the presumed action of lithium is through the regulation of the neurotransmitter glutamate" (2017, p. 323). So the correct answer is b.

B.B. has recently been informed that his wife has died. B.B. refuses to accept his wife's death and cannot be convinced otherwise. When the nurse tries to talk with him about his wife's death he states that he is "too busy to talk" and proceeds to list off what errands he has to get done today. What defense mechanisms is B.B. displaying? Select all that apply. a) Rationalization b) Denial (correct) c) Intellectualization (correct) d) Compensation

Incorrect: A) "Rationalization consists of justifying illogical or unreasonable ideas, actions, or feelings by developing acceptable explanations that satisfy the teller as well as the listener." This defense mechanism is not displayed in this case. Correct: B) "Denial involves escaping unpleasant, anxiety-causing thoughts, feelings, wishes, or needs by ignoring their existence." This is displayed by B.B. refusing to accept that his wife has died. Correct: C) "Intellectualization is a process in which events are analyzed based on remote cold facts and without passion, rather than incorporating feelings and emotion into processing." This is displayed by B.B. focusing on all of his tasks he needs to complete today instead of processing the grief. incorrect: D) "Compensation is used to counterbalance perceived deficiencies by emphasizing strengths." This defense mechanism is unrelated in this scenario.

Corinne Krueger Corinne Krueger Feb 15, 2018 Feb 15 at 6:19pm Manage Discussion Entry All of the following are primary risk factors for depression except: a. Male gender b. Major depressive disorder diagnosis in a first-degree family member c. Adverse experiences during childhood d. Comorbid chronic or disabling conditions

The correct answer is A. According to Halter in Varcarolis' Foundations of Psychiatric Mental Health Nursing, the primary risk factors for depression are: FEMALE gender, adverse childhood experiences, stressful life events, first-degree family members with major depressive disorder, neuroticism, other disorders such as substance use, anxiety, and personality disorders, and chronic or disabling medical conditions. Females are more likely than males to be diagnosed with depression (Halter, 2018, p. 246).

You are a nurse on night shift and you have a patient who has been taking Topiramate (Topamax) to help with their bipolar disorder. Which of the following is the mechanism of action of Topiramate? A. it works by enhancing of gamma-aminobutyrate (GABA), an inhibitory neurotransmitter. B. It works by blocking GABA in the brain. C. It works by increasing serotonin levels in the brain. D. It works by decreasing dopamine in the brain

The correct answer is A. It works by increasing the GABA neurotransmitter in the brain, to keep a person from having huge swings of mood from mania to depression.

Mr. Waters has complaints of dry mouth since starting the medication Risperdal a few weeks ago. What can the nurse tell the patient to help relieve the symptoms of dry mouth? ( Select all that apply) a. encourage ice chips b. chew sugar free gum c. drink frequent sips of water d. drink lemonade

The correct answer will be (a),(b), and (c). According to Halter pg. 214, dry mouth can be a common side effect of the medication. Nursing care can include encouraging ice chips, frequent sips of water, or chew sugar free gum can help with dealing with the symptoms of dry mouth.

Mrs.Smith has been diagnosed with Bipolar I, her physician has recently changed her medication to Lithium. Mrs.Smith started taking Lithium two weeks ago, her recent blood Lithium levels are 1.5 mEq/L(normal therapeutic range 0.6 mEq/L to 1.2 mEq/L). Which of the following symptoms will Mrs. Smith be showing for early signs of toxicity to Lithium? (Select all that apply) a. hand tremors b. confusion c. sedation d. gastrointestinal upset e. all of the above

The correct answer will be (e) all of the above. According to Halter pg. 234. Lithium blood serum level should reach therapeutic level of 0.6 to 1.2 mEq/L. Lithium should not exceed 1.5 mEq/L to avoid serous toxicity. By the lithium level being 1.5mEq/L hand tremors, confusion, sedation, and gastrointestinal upset are early signs of toxicity in addition to hyper irritability of muscles, electroencephalographic changes and incoordination. According to Haler pg. 234 Lithium medication should be withheld, blood lithium levels measured, and dosage reevaluated.

A 45-year-old male was recently diagnosed with depression. As a nurse what would be an appropriate intervention be when caring for this patient. Choose all that apply. a. Provide periods of down time after activities b. When possible, encourage family or friends to remain with the patient during meals c. Allow that patient to sleep as long as they would like throughout the day d. Encourage the use of relaxation measures in the evening

The correct answers are A, B and D. A is correct because fatigue can intensify feelings of depression. B is correct because it reinforces the idea that someone cares and can raise the patient's self-esteem. D is correct because relaxation in the evenings cause induce relaxation and sleep allowing for a better quality sleep. C is not correct because if you minimize sleep during the day it increases the chance of sleep at night and this will help establish a healthy routine.

You are assessing patient A.A. when they state "sometimes at work my heart beating faster and harder and I can't control my breathing. I start sweating and shaking, and I can't think clearly." You recognize these symptoms as? a) mild anxiety b) moderate anxiety (Correct) c) severe anxiety d) panic

a) incorrect. A person experiencing a mild level of anxiety sees, hears, and grasps more information, and problem solving becomes more effective. Physical symptoms may include slight discomfort, restlessness, irritability, or mild tension-relieving behaviors. b) correct. The person experiencing moderate anxiety the ability to think clearly is hampered, but learning and problem solving may take place although not at optimal level. The sympathetic nervous system symptoms begins to kick in. The individual may experience tension, pounding heart, increased pulse and respiratory rate, perspiration. c) incorrect. A person with severe anxiety may focus on one particular detail or many scattered details and have difficulty noticing what is going on in the environment, even when another points it out. Learning and problem solving are not possible at this level, and the person may be dazed and confused. Somatic symptoms often increase; trembling and a pounding heart are common, and the person may experience hyperventilation and a sense of impending doom or dread. d) incorrect. Someone in a state of panic is unable to process what is going on in the environment and may lose touch with reality. The behavior that results may be manifested as pacing, running, shouting, screaming, or withdrawal. Hallucinations, or false sensory perceptions may be experienced. Physical behavior may become erratic, uncoordinated, and impulsive.

D.W. has been experiencing sudden onset of fear of impending doom or dying, increased pulse and respiration's, shortness of breath and dizziness which has lead to a diagnosis of severe anxiety. While discussing some short term goals with her nurse, her nurse suggested ways to help DW gain mastery over panic episodes by suggesting which of the following interventions: (select all that apply) a)Help patient connect feelings before attack with onset of attack: "What were you thinking about just before the attack?" b) Help patient recognize symptoms as resulting from anxiety, not from a catastrophic physical problem. c) Teach patient that if panic episodes continue, to stop taking their medications until they can talk to their health care provider d) Teach patient to use positive self-talk, such as "I can control my anxiety."

a). Physiological symptoms of anxiety usually appear first as the result of a stressor. They are immediately followed by automatic thoughts, such as "I'm dying" or "I'm going crazy," which are distorted assessments. b). Factual information and alternative interpretations can help patient recognize distortions in thought. d) Cognitive restructuring is an effective way to replace negative self-talk.

You a nurse caring for a patient with Bipolar Type 1 taking Lithium 300 mg three times daily. Which of the following signs would be indicative of Lithium toxicity? (Select all that apply) a. vomiting b. muscle weakness c. dry mouth d. slurred speech e. headache

a, b, and d. These are three of the several signs and symptoms listed for lithium toxicity in the davis drug guide as well as twitching, diarrhea, drowsiness, and decreased coordination.

An 89 year old woman comes into the ER with her daughter. The daughter states that her mother has been acting different. She described that her mother has been having mood changes, becomes lethargic then irritated then back to lethargic, and thinks there are spiders crawling on her walls. She was recently discharged after a small surgery that required her to have an indwelling catheter in the hospital. The nurse suspects delirium due to a possible UTI. The 4 Cardinal Signs of delirium are: (check all that apply) a) inattention b) disorganized thinking c) sudden onset d) gradual onset e) fluctuated level of consciousness

a, b, c, and e. According to Giddens (2017), the 4 Cardinal Signs of delirium or acute confusion are sudden onset, fluctuated level of consciousness, inattention, and disorganized thinking. Chronic confusion, or dementia, has a slow, gradual onset.

Which interventions should the nurse use for a patient in a manic state (Choose all that apply)? a. Use short and concise explanations and statements b. Ask only open ended questions to better understand the patient's mental status c. Guide the patient to interact with many people to feel more normal d. Encourage the patient to take frequent naps to promote adequate sleep

a, d Patients in a manic state have a shorter attention span which "limits comprehension to small bits of information" (Halter 231). Additionally, lack of sleep is common in manic patients, and can lead to exhaustion as well as increased mania (Halter 231). B does not apply because as mentioned above, patients with mania have short attention spans, and using only open ended questions/statements will likely lead the patient down a flight of ideas, revealing little about their mental status. Instead, concise questions should be used, either exclusively or in combination with a few open ended questions (Halter 231). C. Is not true because according to Halter (231), patients should be placed in a low stimulus environment. "Relaxation is promoted, and manic behavior is minimized" (Halter 231).

Ms. Buttercup has been taking mood stabilizers for she asked her doctor if there were other alternatives to taking mood stabilizers? What where some the choices the doctor would have gave her? (choose all that apply) A. Talking treatments B. Keep a mood diary C. Look after your general health D. Try complementary therapies E. Peer support F. Discuss a different medication with your doctor

a,b,c,d,e,f it is all of them they are all following options that could work for Ms. Buttercup. The National Institute for Health and Care Excellence advises to due this it can lead to help identify any patterns in her cognitive behavioral therapy. It allows for Ms. Buttercup to be involved in her treatment and to have a voice.

Ms. Buttercup is diagnosed with anxiety. She has to give a speech today. She is worried and looking for the fastest way out of the room if she has to exit. She often has headaches and thinks she is going to die. What drug would be prescribed to Ms. Buttercup to increase levels of GABA and to decrease the frequency of here migraine headaches? A. Depakote B. Lorazepam C. Abilify D. Imitrex

a. This is a mood stabilizer medication it increases the levels of GABA an inhibitory neurotransmitter in the CNS. It also works in the prevention of migraine headache. The answer b is wrong due to it is a sedative it does decrease anxiety but is use more as a sedation for surgery. The answer c. is wrong due to its an antipsychotic it is used mainly for schizophrenia and bipolar disorder. The answer d. is wrong it is used for migraine headaches but has nothing to do with anxiety.

According the DSM-5, a severe panic disorder is characterized by all of the following symptoms except a)trembling or shaking b)sensations of shortness of breath or smothering c)derealization d)akathisia

d (Halter 276) Akathisia a movement disorder characterized by a feeling of inner restlessness and inability to stay still. Although a panic episode can have feelings of restlessness, it is not listed by the DSM-5 as a require symptom for diagnosis.

Ms. Buttercup is taking Depakote for anxiety. She states, "I don't like the way it makes me feel when I take it." You ask how does it make you feel? She proceeds to tell you she has abdominal pain, diarrhea, with nausea and vomiting but with an increased appetite. What is Ms. Buttercup experiencing? A. Stomach ulcers caused by the Depakote B. Depakote causing renal impairment which is producing these side effects C. Drug reaction with eosinophilia and systemic symptoms D. Hepatotoxicity

d. It is that the drug is not right for her or the dose is too high. It will cause abdominal pain, diarrhea, associated with n/v and increased appetite. It becomes a vicious cycle of eating and diarrhea. B. is wrong it does not cause renal impairment but should be used cautiously in pts with renal impairment. C. is wrong due to Depakote can cause a skin reaction but the question is about GI. A. is wrong because Depakote does not cause stomach ulcers.

A patient in the ICU needs to be assessed for delirium. The nurse knows that the best way assess for delirium is the Confusion Assessment Method (CAM). Which of the following are appropriate questions to ask for the CAM test? (select all that apply) a) "Do you have a history of hypertension?" b) "repeat the alphabet from A-Z." c) "squeeze my hand when I say the letter 'A'." d) "Does a stone float on water?"

the CAM test is used to assess patients with delirium. Good questions to ask are ones that challenge the patient's thinking.

You are a nurse caring for a 67 year old female patient who is recovering from a surgery she has earlier in the day. When she wakes up she has reduced ability to direct, focus, shift, and sustain attention. She also has disorganized thinking and disturbance of consciousness. What is the most likely diagnosis? a) Alzheimer's b) acute onset delirium c) schizophrenia d) brain tumor

there are four cardinal features of delirium 1. Acute onset and fluctuating course 2. Reduced ability to direct, focus, shift, and sustain attention 3. Disorganized thinking 4. Disturbance of consciousness.


Set pelajaran terkait

ACCT 2100 Syllabus Spring 2020 Qu- Exam 1 Study Guide

View Set

Renal Calculi NCLEX style questions

View Set

Experimental Design & Normal Distributions

View Set

Fundies Chapter 41: Self-Concept PrepUs

View Set

Chapter 10 Healthcare Delivery System

View Set

Ch. 16-19 Medical-Surgical Review Questions

View Set

Anatomy Day 5 Boney Landmarks and Structure of the Vertebral Column

View Set